Adam Sullivan Robbins and Cotran Path Basis of Disease Qs Flashcards

1
Q

A 38-year-old man has had headaches and nausea for the past 2 months. Laboratory findings show hypercalcemia and hypophosphatemia and normal serum albumin. Urine microscopic analysis shows deposition of calcium salts in the renal tubular epithelium. Which of the following processes has most likely produced this change in the kidney?

Dystrophic calcification
Renal tubular atrophy
Autophagocytosis
Metastatic calcification
Cellular aging
A

Metastatic calcification

Deposition of calcium in normal healthy tissues as a result of prolonged hypercalcemia is called metastatic calcification. This process may occur in hyperparathyroidism. Dystrophic calcification refers to calcium deposition in injured tissues, with normal serum calcium levels. Atrophy decreases cell size but is not accompanied by calcium deposition. Autophagocytosis yields more golden-brown lipofuscin pigment in the cytoplasm, particularly in hepatocytes and myocardial fibers, a process that becomes more apparent with aging

How well did you know this?
1
Not at all
2
3
4
5
Perfectly
2
Q

An experiment introduces a “knockout” gene mutation into a cell line. The frequency of cell karyorrhexis and cytoplasmic fragmentation is increased, compared with a cell line without the mutation. Overall survival of the mutant cell line is reduced. Which of the following genes is most likely to be affected by this mutation?

BAX
BCL-2
C-MYC
FAS
TP53
A

BCL-2

The BCL-2 gene product inhibits cellular apoptosis by binding to Apaf-1. The BAX gene product promotes apoptosis. The C-MYC gene is involved with oncogenesis. The FAS gene encodes for a cellular receptor for FAS ligand, which signals apoptosis. TP53 gene activity normally stimulates apoptosis, but mutation favors cell survival.

Adam notes - BCL-2 mutation found in t(14;18) of Follicular lymphoma. Translocation with IgH.

How well did you know this?
1
Not at all
2
3
4
5
Perfectly
3
Q

A 43-year-old man has had a cough and fever for the past 2 months. A chest radiograph shows bilateral nodular densities, some with calcification, located mainly in the upper lobes of the lungs. A transbronchial lung biopsy is performed, yielding a specimen with the microscopic appearance showing a granuloma with many epithelioid cells and prominent large Langhans giant cells. Which of the following chemical mediators is most important in the pathogenesis of this lesion?

Complement C5a
Interferon-γ
Bradykinin
Nitric oxide
Prostaglandin
A

IFN gamma

Macrophage stimulation and transformation to epithelioid cells and giant cells are characteristic of granuloma formation. Interferon-γ promotes the formation of epithelioid cells and giant cells. Complement C5a is chemotactic for neutrophils. Although occasional neutrophils are seen in granulomas, neutrophils do not form a major component of granulomatous inflammation. Bradykinin, released in acute inflammatory responses, results in pain. Macrophages can release nitric oxide to destroy other cells, but nitric oxide does not stimulate macrophages to form a granulomatous response. Prostaglandins are mainly involved in the causation of vasodilation and pain in acute inflammatory responses.

How well did you know this?
1
Not at all
2
3
4
5
Perfectly
4
Q

A 55-year-old woman has had discomfort and swelling of the left leg for the past week. On physical examination, the leg is slightly difficult to move, but on palpation, there is no pain. A venogram shows thrombosis of deep left leg veins. Which of the following mechanisms is most likely to cause this condition?

Turbulent blood flow
Nitric oxide release
Ingestion of aspirin
Hypercalcemia
Immobilization
A

Immobilisation

The most important and the most common cause of venous thrombosis is vascular stasis, which often occurs with immobilization. Turbulent blood flow may promote thrombosis, but this risk factor is more common in fast-flowing arterial circulation. Nitric oxide is a vasodilator and an inhibitor of platelet aggregation. Aspirin inhibits platelet function and limits thrombosis. Calcium is a cofactor in the coagulation pathway, but an increase in calcium has minimal effect on the coagulation process.

How well did you know this?
1
Not at all
2
3
4
5
Perfectly
5
Q

A 70-year-old man who was hospitalized 3 weeks ago for a cerebral infarction is ambulating for the first time. Within minutes of returning to his hospital room, he has sudden onset of dyspnea with diaphoresis. He cannot be resuscitated. Which of the following risk factors most likely contributed to this finding?

Venous stasis
Pulmonary arterial atherosclerosis
Lupus anticoagulant
Bronchopneumonia
Factor V mutation
A

Venous stasis

The figure shows a large pulmonary thromboembolus. The most common risk factor is immobilization leading to venous stasis. These thrombi form in the large deep leg or pelvic veins, not in the pulmonary arteries. Coagulopathies from acquired or inherited disorders, such as those from lupus anticoagulant (antiphospholipid antibodies) or factor V (Leiden) mutation, are possible causes of thrombosis, but they usually manifest at a younger age. These causes also are far less common risks for pulmonary thromboembolism than is venous stasis. Local inflammation from pneumonia may result in thrombosis of small vessels in affected areas.

How well did you know this?
1
Not at all
2
3
4
5
Perfectly
6
Q

A 25-year-old woman with amenorrhea has never had menarche. On physical examination, she is 145 cm (4 ft, 9 in) tall. She has a webbed neck, a broad chest, and widely spaced nipples. Strong pulses are palpable in the upper extremities, but there are only weak pulses in the lower extremities. On an abdominal MR imaging, the ovaries are, the ovaries are small, elongated, and tubular. Which of the following karyotypes is most likely to be present in this patient?

45,X/46,XX
46,X,X(fra)
47,XXY
47,XXX
47,XX,+16
A

45,X/46,XX

The features described are those of classic Turner syndrome. Individuals who reach adulthood may have mosaic cell lines, with some 45,X cells and some 46,XX. A female carrier of the fragile X syndrome, X(fra), is less likely to manifest the disease than a male, but the number of triple repeat sequences (CGG) increases in her male offspring. The 47,XXY karyotype occurs in Klinefelter syndrome; affected individuals appear as phenotypic males. The “superfemale” karyotype (XXX) leads to mild.

How well did you know this?
1
Not at all
2
3
4
5
Perfectly
7
Q

A 10-year-old boy who is mentally retarded is able to carry out activities of daily living, including feeding and dressing himself. On physical examination, he has brachycephaly and oblique palpebral fissures with prominent epicanthal folds. A transverse crease is seen on the palm of each hand. On auscultation of the chest, there is a grade III/VI systolic murmur. Which of the following diseases is he most likely to develop by age 20?

Acute leukemia
Hepatic cirrhosis
Chronic renal failure
Acute myocardial infarction
Aortic dissection
A

Acute leukemia

This boy has Down syndrome (trisomy 21), one of the trisomies that can result in a liveborn infant. Although children with Down syndrome can function fairly well, they often have many associated congenital anomalies. Among the more common is congenital heart disease, including ventricular septal defect. There is also a 10- to 20-fold increased risk of acute leukemia. Virtually all individuals with Down syndrome who live to the age of 40 will have evidence of Alzheimer disease. Hepatic cirrhosis is a feature of galactosemia. Chronic renal failure may be seen in genetic disorders that produce polycystic kidneys. Myocardial infarction at a young age suggests familial hypercholesterolemia. Aortic dissection is seen in persons with Marfan syndrome.

How well did you know this?
1
Not at all
2
3
4
5
Perfectly
8
Q

A 25-year-old woman gives birth to a term infant with a right transverse palmar crease, low-set ears, oblique palpebral fissures, and a heart murmur. The infant survives to childhood and exhibits only mild mental retardation. Which of the following chromosomal abnormalities is most likely to be present in this child?

Haploidy
Monosomy
Mosaicism
Tetraploidy
Triploidy
A

Mosaicism

These features are characteristic of trisomy 21, but the child is not severely affected, which suggests mosaicism. In mosaic individuals, greater numbers of potentially normal cells having the proper chromosomal complement are present, which may allow infants with abnormalities of chromosome number to survive to term and beyond. Haploidy is present in gametes. Loss of an autosomal chromosome is devastating; the only monosomy associated with possible survival to term is Turner syndrome (monosomy X). Most aneuploid conditions (trisomies and monosomies) lead to fetal demise; fetuses with trisomy 21 are the most likely to survive to term. Triploid fetuses rarely survive beyond the second trimester and are virtually never liveborn. Likewise, tetraploidy accounts for many first-trimester fetal losses and is not survivable.

How well did you know this?
1
Not at all
2
3
4
5
Perfectly
9
Q

A 30-year-old woman has had fever and arthralgia for the past 2 weeks. On physical examination, she has a temperature of 37.6°C and an erythematous malar rash. Initial laboratory studies are positive for ANAs at 1:1600 and anti–double-stranded DNA antibodies at 1:3200. Serum creatinine is markedly elevated, and serum complement levels are decreased. A VDRL test for syphilis is positive, and in vitro tests of coagulation (prothrombin time and partial thromboplastin time) are prolonged. Which of the following clinical features of her illness is most likely caused by antibodies that interfere with the coagulation test?

Arthritis
Recurrent thrombosis
Rash
Renal failure
Fever
A

Recurrent thrombosis

This patient has clinical and serologic features of systemic lupus erythematosus (SLE). She also has a false-positive test result for syphilis, indicating the presence of anticardiolipin antibodies. These antibodies against phospholipid-protein complexes (antiphospholipid antibodies) are also called lupus anticoagulants, because they interfere with in vitro clotting tests. However, in vivo, they are thrombogenic. Hence, these patients can have recurrent thrombosis. Lupus anticoagulants can also occur in the absence of lupus. The other listed options can occur in SLE, but they are not mediated by antiphospholipid antibodies.

How well did you know this?
1
Not at all
2
3
4
5
Perfectly
10
Q

A 37-year-old man who is HIV positive has noticed multiple 0.5- to 1.2-cm, plaque-like, reddish-purple skin lesions on his face, trunk, and extremities. Some of the larger lesions appear to be nodular. These lesions have appeared over the past 6 months and have slowly enlarged. Molecular analysis of the spindle cells found in these skin lesions is likely to reveal the genome of which of the following viruses?

Cytomegalovirus
Epstein-Barr virus
Adenovirus
Human herpesvirus-8
HIV-1
A

HHV8

This patient has AIDS, with Kaposi sarcoma of the skin. Kaposi sarcoma is associated with a herpesvirus agent that is sexually transmitted: human herpesvirus 8 (HHV-8), also called the Kaposi sarcoma herpesvirus. Other herpesviruses are not involved in the pathogenesis of Kaposi sarcoma, although infection with these viruses can occur frequently in persons with AIDS. HIV, although present in the lymphocytes and monocytes, is not detected in the spindle cells that proliferate in Kaposi sarcoma. With the exception of the varicella-zoster virus, which is associated with dermatomally distributed skin vesicles known as shingles, skin lesions are not common manifestations of herpesviruses, which include cytomegalovirus, Epstein-Barr virus, or adenovirus infections.

Adam notes - HHV8 also associated with Castleman’s disease: multicentric angiofollicular lymph node hyperplasia

How well did you know this?
1
Not at all
2
3
4
5
Perfectly
11
Q

A 45-year-old woman has experienced difficulty in swallowing that has increased in severity over the past year. She has also experienced malabsorption, demonstrated by a 5-kg weight loss in the past 6 months. She reports increasing dyspnea during this time. On physical examination, her temperature is 36.9°C, pulse 66/min, respirations 18/min, and blood pressure 145/90 mm Hg. Echocardiography shows a large pericardial effusion. The ANA test result is positive at 1:512 with a nucleolar pattern. Which of the following serious complications of the patient’s underlying autoimmune disease is most likely to occur?

Meningitis
Glomerulonephritis
Perforated duodenal ulcer
Adrenal failure
Malignant hypertension
A

Malignant hypertension

This patient has diffuse systemic sclerosis (scleroderma). The small arteries of the kidney are damaged by a hyperplastic arteriolosclerosis that can be complicated by very high blood pressure and renal failure. Meningitis and adrenal failure are not typical features of autoimmune diseases. Glomerulonephritis is a more typical complication of systemic lupus erythematosus. With scleroderma, the gastrointestinal tract undergoes fibrosis, without any tendency to perforation or ulceration.

Adam notes: diffuse scleroderma associated with anti-topoisomerase (SCL-70) autoAb

How well did you know this?
1
Not at all
2
3
4
5
Perfectly
12
Q

An epidemiologic study is conducted to determine risk factors for HIV infection. The study documents that persons with coexisting sexually transmitted diseases such as chancroid are more likely to become HIV positive. It is postulated that an inflamed mucosal surface is an ideal location for the transmission of HIV during sexual intercourse. Which of the following cells in these mucosal surfaces is most instrumental in transmitting HIV to CD4+ T lymphocytes?

CD8+ cells
Natural killer cells
Dendritic cells
Neutrophils
Plasma cells
A

Dendritic cell

Three types of cells can carry HIV: dendritic cells, monocytes, and CD4+ T cells. Mucosal dendritic cells (i.e., Langerhans cells) can bind to the virus and transport it to CD4+ cells in the lymph nodes. Whether the virus is internalized by mucosal dendritic cells is not clear. Monocytes and CD4+ T cells express CD4 and the coreceptors (CCR5 and CXCR4); therefore, HIV can enter these cells. Follicular dendritic cells are distinct from mucosal or epithelial dendritic cells; they trap antibody-coated HIV virions by means of their Fc receptors. The other listed cells cannot be infected by HIV

How well did you know this?
1
Not at all
2
3
4
5
Perfectly
13
Q

A 26-year-old woman has had bouts of joint pain for the past 2 years. She also has a rash on the cheeks and bridge of the nose. On physical examination, there is no joint swelling or deformity, although generalized lymphadenopathy is present. Laboratory studies indicate anemia, leukopenia, a polyclonal gammopathy, and proteinuria. The serum ANA test result is positive at a titer of 1:1024 with a rim pattern identified by immunofluorescence. The light microscopic and immunofluorescent (with antibody to IgG) appearances of a skin biopsy are shown. Which of the following is the best information to give this patient about her disease?

Blindness is likely to occur within 5 years
Avoid exposure to cold environments
Joint deformities will eventually occur
Chronic renal failure is likely to occur
Cardiac valve replacement will eventually be required

A

Chronic renal failure

This patient has systemic lupus erythematosus (SLE). Many persons with SLE have glomerulonephritis and eventually develop renal failure. Blindness is uncommon in SLE. Raynaud phenomenon is associated with many autoimmune diseases, but it is most troublesome in scleroderma. Although synovial inflammation is common in SLE, joint deformity is rare. The Libman-Sacks endocarditis associated with SLE tends to be nondeforming and limited, and there is minimal valve damage. It is now uncommon, because of the use of corticosteroid therapy in the treatment of SLE.

Adam notes: She fulfils the diagnostic criteria for SLE having at least 4 of the 11 of SOAP BRAIN MD. She has arthralgia, malar rash, blood dyscrasias, ANA, and renal disease.

How well did you know this?
1
Not at all
2
3
4
5
Perfectly
14
Q

A 9-month-old child has a history of recurrent infections with multiple agents, including cytomegalovirus, Candida albicans, Staphylococcus aureus, and Staphylococcus epidermidis. A careful family history and pedigree analysis show this to be a genetic disorder that is inherited in an autosomal recessive pattern. Which of the following laboratory studies is likely to be most useful in establishing the underlying mechanism of immunodeficiency in this infant?

Quantitative serum immunoglobulin levels
Enumeration of B cells in blood
Enumeration of CD3+ cells in blood
Tests of neutrophil function
Adenosine deaminase levels in leukocytes
A

Adenosine deaminase levels in leukocytes

This patient is susceptible to bacterial, fungal, and viral infections and most likely has severe combined immunodeficiency (SCID). The autosomal recessive pattern of inheritance implicates adenosine deaminase (ADA) deficiency rather than mutations in the γ chain of cytokine receptors. Low ADA levels in the leukocytes are diagnostic. The other listed options are relevant to the workup of primary immunodeficiencies, but they are not specific to SCID.

How well did you know this?
1
Not at all
2
3
4
5
Perfectly
15
Q

A 28-year-old man has had hemoptysis and hematuria for the past 2 days. On physical examination, his temperature is 36.8°C, pulse 87/min, respirations 19/min, and blood pressure 150/90 mm Hg. Laboratory studies show creatinine of 3.8 mg/dL and urea nitrogen of 35 mg/dL. Urinalysis shows 4+ hematuria, 2+ proteinuria, and no glucose. A renal biopsy specimen examined microscopically shows glomerular damage and linear immunofluorescence with labeled anti-complement and anti-IgG antibody. Which of the following autoantibodies has the greatest specificity for this patient’s condition?

Anti–double-stranded DNA
Antihistone
Anti–Jo-1
Anti–U1-riboneucleoprotein
Anti–SS-A
Anti–basement membrane
Anti-phospholipid
A

Anti-basement membrane

This patient has Goodpasture syndrome, in which an antibody is directed against type IV collagen in basement membranes of the glomeruli and in the lung. This is a form of type II hypersensitivity reaction. The antibodies attach to the basement membrane and fix complement, thus damaging the glomeruli. Anti–double-stranded DNA antibodies have specificity for systemic lupus erythematosus (SLE), whereas antihistone antibodies are characteristic of drug-induced SLE. Anti–Jo-1 antibody is found in dermatomyositis and polymyositis. The anti-U1-ribonucleoprotein antibody is seen in mixed connective tissue diseases. Anti–SS-A antibody is seen in Sjögren syndrome. Antiphospholipid antibodies are sometimes called “lupus anticoagulant,” because they may appear in SLE; such patients have coagulopathies with thrombosis or bleeding, or both.

How well did you know this?
1
Not at all
2
3
4
5
Perfectly
16
Q

A 61-year-old man has had increasing malaise for the past 4 months. On physical examination, he is afebrile and has mild muscle wasting. Laboratory studies show serum creatinine of 4.5 mg/dL and urea nitrogen of 44 mg/dL. Urine dipstick analysis shows no blood, protein, or glucose, but a specific test for Bence Jones proteins yields a positive result. A renal biopsy specimen has the microscopic appearance shown in the figure = amyloidosis. Which of the following underlying conditions is most likely to be present in this patient?

Rheumatic fever
Multiple myeloma
Ankylosing spondylitis
Systemic sclerosis
Common variable immunodeficiency
A

Multiple myeloma

Amyloidosis is most often caused by excessive light chain production with plasma cell dyscrasias such as multiple myeloma (AL amyloid). Chronic inflammatory conditions, such as rheumatic fever, ankylosing spondylitis, and systemic sclerosis, may also result in amyloidosis (AA amyloid), but not in secretion of light chains in urine (i.e., Bence Jones proteinuria). Immunoglobulin levels are generally reduced in patients with common variable immunodeficiency.

How well did you know this?
1
Not at all
2
3
4
5
Perfectly
17
Q

A 66-year-old woman sees her physician because of a lump on the right side of the neck that has been increasing in size for the past 7 months. On physical examination, the physician palpates a firm but nontender 3-cm mass in the posterior cervical lymph node. Microscopic examination of a biopsy specimen of the mass shows effacement of the nodal architecture by a monomorphous population of large cells with large, dark blue nuclei and scant cytoplasm. The peripheral blood smear and bone marrow biopsy results are normal. Which of the following is the most likely diagnosis?

Lymphangioma
Reactive hyperplasia
Non-Hodgkin lymphoma
Multiple myeloma
Chronic myelogenous leukemia
A

Non-Hodgkin’s Lymphoma

Non-Hodgkin lymphomas are malignant neoplasms of lymphoid tissues. They have no benign equivalent. Monomorphous proliferations that destroy the nodal architecture suggest a neoplasm. A lymphangioma is composed mostly of a proliferation of lymphatics. Reactive hyperplasias are polyclonal, controlled proliferations with a diverse cell population. A myeloma is composed of plasma cells and most often involves bone marrow. A leukemia is a neoplasm that arises in the bone marrow and spills over into peripheral blood.

How well did you know this?
1
Not at all
2
3
4
5
Perfectly
18
Q

During a routine health maintenance examination, a 46-year-old man is found to have an enlarged, nontender supraclavicular lymph node that is palpable on physical examination. The 2-cm node is excised. Histologically, the nodal architecture is effaced by a monomorphous population of small lymphocytes. Which of the following procedures would best confirm that the patient has a malignancy?

Peripheral WBC count and differential cell count
Flow cytometry of nodal tissue for DNA content
Electron microscopy to determine cellular ultrastructure
Southern blot analysis to demonstrate monoclonality
Determination of the serum lactate dehydrogenase level

A

Southern blot analysis to demonstrate monoclonality.

Monoclonality is the hallmark of a malignancy. In the diagnosis of a leukemia, the WBC count is helpful but not definitive. The DNA content analysis alone cannot define a malignancy; Southern blot analysis for T- or B-cell receptor gene rearrangements can define monoclonality. Electron microscopy is an adjunct to diagnosis of the type of tumor. Lactate dehydrogenase levels are often increased with lymphoid proliferations but are not diagnostic of the type of proliferation. See pages 278, 324-327.

How well did you know this?
1
Not at all
2
3
4
5
Perfectly
19
Q

An epidemiologic study is performed to assess risks for cervical carcinoma. The cells from cervical lesions in a population of women are analyzed, and there is viral binding to pRB in patients in whom dysplastic cells are present. Which of the following viruses is most likely to bind to pRB to increase the risk for these lesions?

Cytomegalovirus
Epstein-Barr virus
Herpes simplex virus
Hepatitis B virus
HIV
Human papillomavirus
JC papovavirus
A

HPV

Human papillomavirus (HPV) types 16, 18, and 31 encode proteins that bind TP53 with high affinity, resulting in loss of tumor suppressor activity. Seventy-five to nearly 100% of squamous epithelial dysplasias and carcinomas of the cervix are associated with HPV infection. Cytomegalovirus and herpes simplex virus do not participate directly in carcinogenesis. Epstein-Barr virus is associated with some malignant lymphomas and nasopharyngeal carcinomas. Hepatitis B virus is associated with hepatocellular carcinomas arising in the setting of regeneration in chronic liver injury. HIV does not affect pRB, but the loss of immune regulation promotes development of lymphomas and Kaposi sarcoma. The JC papovavirus is associated with development of progressive multifocal leukoencephalopathy.

Adam notes - HPV proteins E6 binds p53, E7 binds Rb causing these TSGs to become non-functional.

How well did you know this?
1
Not at all
2
3
4
5
Perfectly
20
Q

A 23-year-old woman has noted a nodule on the skin of her upper chest. She reports that the nodule has been present for many years and has not changed in size. On physical examination, there is a 0.5-cm dark-red, nontender, raised nodule with a smooth surface. Which of the following is the most likely diagnosis?

Adenoma
Fibroadenoma
Hamartoma
Hemangioma
Leiomyoma
Lipoma
Melanoma
Nevus
A

Haemangioma

The small, discrete nature of this mass and its relatively unchanged size suggest a benign neoplasm. The red color suggests vascularity. A hemangioma is a common benign lesion of the skin. Adenomas arise in glandular epithelium, such as the colon. Fibroadenomas arise in the breast. A hamartoma contains a mixture of cell types common to a tissue site. Leiomyomas, which are white, arise from smooth muscle and are most common in the uterus. Lipomas are yellow fatty tumors. Melanomas are malignant and tend to increase in size quickly; many are darkly pigmented. The benign counterpart to the melanoma is the nevus, which is quite common; nevi are usually light brown.

How well did you know this?
1
Not at all
2
3
4
5
Perfectly
21
Q

A 70-year-old man has sprayed his orchard with insecticide each spring for 20 years. He recently noticed a rough, erythematous area of skin on the right shoulder. The patch becomes ulcerated and does not heal. He develops weight loss, nausea, and vomiting. The excised lesion is a squamous cell carcinoma. Which of the following was most likely a component of the insecticide that was used by this man?

Arsenic
Benzene
Cadmium
Chromium
Chlorinated biphenyl
Ethylene oxide
Naphthalene
A

Arsenic

Arsenic can cause skin cancer. However, lead arsenate has not been in widespread use for years, and occupational safety measures have reduced risks to workers from the use of chemicals in agriculture and industry. Benzene is linked to leukemias. Cadmium exposure is linked to prostate cancer (dispose of old batteries properly!). Some lung cancers are linked to chromium exposure. Chlorinated biphenyls, contained in many insect sprays, have not been linked directly to cancer. Ethylene oxide exposure carries an increased risk of leukemia. Ethylene oxide has been employed as a ripening agent for some agricultural products, and it is also used as a disinfectant gas for surgical equipment in hospitals. Exposure to naphthalene compounds is a risk factor for urinary tract cancer.

How well did you know this?
1
Not at all
2
3
4
5
Perfectly
22
Q

A 38-year-old woman has abdominal distention that has been worsening for the past 6 weeks. An abdominal CT scan shows bowel obstruction caused by a 6-cm mass in the jejunum. At laparotomy, a portion of the small bowel is resected. Microscopic examination shows that the mass is a Burkitt lymphoma. Flow cytometry analysis of a portion of the tumor shows a high S phase. Mutational activation of which of the following nuclear oncogenes is most likely to be present in this tumor?

ERBB2
TP53
RAS
MYC
APC
A

MYC

The MYC oncogene is commonly activated in Burkitt lymphoma because of a t(8;14) translocation. The MYC gene binds DNA to cause transcriptional activation of growth-related genes such as that for cyclin D1, resulting in activation of the cell cycle. ERBB2 (also known as HER2) encodes growth factor receptor located on the cell surface. TP53 and APC are tumor suppressor genes that are inactivated in many cancers, including colon cancer (APC). RAS oncogene encodes a GTP-binding protein that is located under the cell membrane.

How well did you know this?
1
Not at all
2
3
4
5
Perfectly
23
Q

A 33-year-old woman undergoes a routine physical examination as part of her health maintenance screening. There are no abnormal findings. A Pap smear is obtained as part of the pelvic examination. Cytologically, the cells obtained on the smear from the cervix show severe epithelial dysplasia. Which of the following statements best explains the significance of these findings?

The lesion could progress to invasive cervical carcinoma
An ovarian teratoma is present
There has been regression of a cervical carcinoma
Antibiotic therapy will cure the lesion
Female relatives are at risk of acquiring the same condition

A

The lesion could progress to invasive cervical carcinoma.

Epithelial dysplasias, especially those that are severe, can be precursors of carcinomas. This is a key reason for Pap smear screening; the incidence of cervical carcinoma decreases when routine Pap smears are performed. Teratomas show well-differentiated elements derived from all the germ cell layers, and they do not manifest as epithelial dysplasias. Severe dysplasias are not amenable to antibiotic therapy. Cervical dysplasias are not hereditary. Regression of a malignancy is a rare event.

How well did you know this?
1
Not at all
2
3
4
5
Perfectly
24
Q

A 25-year-old soldier stationed in the Middle East has experienced abdominal enlargement and a 7-kg weight loss over the past 7 weeks. On physical examination, there is hyperpigmentation of skin, hepatosplenomegaly, and generalized lymphadenopathy. Laboratory studies show hemoglobin of 10.0 g/dL, hematocrit 29.9%, platelet count 78,200/mm3, and WBC count 3210/mm3. One month later, he develops a high fever, and Streptococcus pneumoniae is cultured from his blood. Which of the following infectious organisms is most likely to produce these findings?

Borrelia recurrentis
Brugia malayi
Leishmania donovani
Listeria monocytogenes
Mycobacterium leprae
Plasmodium falciparum
Trypanosoma gambiense
A

Leishmania donovani

This patient has visceral leishmaniasis, or kala-azar. Leishmaniasis is endemic in the Middle East, South Asia, Africa, and Latin America. The organisms proliferate within macrophages in the mononuclear phagocyte system and cause hepatosplenomegaly and lymphadenopathy. Often, there is hyperpigmentation of the skin. Bone marrow involvement and splenic enlargement contribute to reduced production and accelerated destruction of hematopoietic cells, giving rise to pancytopenia. Leishmania donovani is transmitted by sand flies and leads to infection of macrophages, which produces hepatosplenomegaly, lymphadenopathy, and bone marrow involvement with pancytopenia.

Borreliosis causes relapsing fever and is transmitted via body lice.

Brugia malayi is a nematode transmitted by mosquitoes that leads to filariasis involving lymphatics to produce elephantiasis.

Listeriosis is most often acquired via contaminated food or water. In most adults, it produces mild diarrheal illness, but in some adults and children, and in fetuses, it may produce meningitis or dissemination with microabscess (microgranuloma) formation.

Mycobacterium leprae causes Hansen disease, with infection of peripheral nerves and skin. In persons with a strong immune response, the tuberculoid form of this disease results in granuloma formation; in those with a weak immune response, the lepromatous form occurs, characterized by large numbers of macrophages filled with short, thin, acid-fast bacilli.

Malaria, caused by Plasmodium falciparum, produces hemolytic anemia, splenomegaly, and cerebral thrombosis.

African trypanosomiasis produces sleeping sickness.

How well did you know this?
1
Not at all
2
3
4
5
Perfectly
25
Q

A 31-year-old man has had increasing respiratory difficulty for the past 2 days. On physical examination, crackles are auscultated over all lung fields. A chest radiograph shows bilateral interstitial infiltrates. Laboratory studies show that the patient is HIV positive and has a plasma titer of 26,800 copies of HIV-1 RNA/mL. A transbronchial biopsy is performed, and the microscopic appearance of the specimen is showing an owl’s eye inclusion. On the basis of the clinical and histologic findings, which of the following is the most likely causative organism of this acute illness?

Epstein-Barr virus
Cytomegalovirus
Respiratory syncytial virus
Herpes zoster virus
Adenovirus
A

CMV

This patient has high HIV-1 RNA levels that are consistent with the diagnosis of AIDS. Although patients with AIDS are susceptible to many microbes, infections with cytomegalovirus (CMV) are particularly common. The biopsy specimen shows an enlarged cell containing a distinct intranuclear inclusion and ill-defined cytoplasmic inclusions, which are typical of CMV infection.

Epstein-Barr virus infection frequently is seen in patients with HIV infection, but there are no distinct pulmonary lesions associated with it.
Respiratory syncytial virus infections are seen in children but rarely in adults.
Herpes zoster infections are most likely to affect the peripheral nervous system, rarely can become disseminated to affect the lungs in immunosuppressed patients, and produce a different appearance than that shown.
Adenovirus is a common viral pathogen in adults that may produce a clinically significant pneumonia, and intranuclear inclusions may be present, but the cells are not large, and cytoplasmic inclusions are absent.

How well did you know this?
1
Not at all
2
3
4
5
Perfectly
26
Q

A 60-year-old man has had persistent bloody diarrhea, abdominal cramps, and fever for the past week. On physical examination, his temperature is 38.1°C. He has mild diffuse abdominal pain. A stool sample is positive for occult blood. Sigmoidoscopic examination shows mucosal ulceration in the cecum and ascending colon. Microscopic examination of a colonic biopsy specimen shows flask-shaped mucosal ulcers with extensive necrosis and a modest, nonspecific, inflammatory response. The ulcers do not penetrate the muscularis propria. Which of the following infectious organisms is most likely to produce these findings?

Giardia lamblia
Entamoeba histolytica
Shigella flexneri
Salmonella enteritidis
Vibrio cholerae
Bacillus cereus
A

Entamoeba histolytica

Amebiasis is a common cause of dysentery in developing nations. The Entamoeba histolytica trophozoites can attach to colonic epithelium, invade, and lyse the epithelial cells. In some cases, there can be extensive mucosal involvement with characteristic flask-shaped (like an Erlenmeyer flask) ulcerations similar to those seen in other severe inflammatory bowel diseases.

Giardiasis tends to involve the small intestine and produces variable inflammation but not ulceration. Shigellosis can produce bloody dysentery with irregular superficial colonic mucosal ulceration, but the organisms typically do not invade beyond the lamina propria. Salmonellosis more typically involves the small intestine and in most cases produces self-limited enteritis, although more severe disease with dissemination to other organs can occur with Salmonella typhi infection. Cholera is characterized by massive, secretory diarrhea without intestinal mucosal invasion or necrosis. Bacillus cereus is a cause of food poisoning (most often as a contaminant in reheated fried rice) and has a short incubation time.

How well did you know this?
1
Not at all
2
3
4
5
Perfectly
27
Q

For the past 3 days, a 68-year-old woman has had a fever and a cough productive of yellow sputum. On physical examination, there is dullness to percussion at the left lung base. A chest radiograph shows areas of consolidation in the left lower lobe. Despite antibiotic therapy, the course of the disease is complicated by abscess formation, and she dies. At autopsy, there is a bronchopleural fistula surrounded by a pronounced fibroblastic reaction. Small, yellow, 1- to 2-mm “sulfur granules” are grossly visible within the area of abscess formation. Which of the following organisms is most likely to produce these autopsy findings?

Actinomyces israelii
Blastomyces dermatitidis
Chlamydia pneumoniae
Klebsiella pneumoniae
Mycobacterium kansasii
A

Actinomyces israelii

Actinomycetes that can produce chronic abscessing pneumonia, particularly in immunocompromised patients, include Actinomyces israelii and Nocardia asteroides. Sulfur granules, formed from masses of the branching, filamentous organisms, are more likely to be seen in Actinomyces.

Blastomyces dermatitidis infections tend to produce a granulomatous inflammatory process. Chlamydial infections produce an interstitial pattern like that of most viruses. Klebsiella infections, like other bacterial infections, can result in abscess formation, although without distinct sulfur granules. Mycobacterium kansasii infections are similar to M. tuberculosis infections in that granulomatous inflammation is prominent.

How well did you know this?
1
Not at all
2
3
4
5
Perfectly
28
Q

A 24-year-old college student comes to the health service because he has had a cough, fever, and shortness of breath for the past 3 weeks when walking. The results of cardiac examination are normal, but crackles are heard in both lungs. A chest radiograph shows patchy infiltrates in the lungs. Laboratory studies show an elevated cold agglutinin titer. A presumptive clinical diagnosis of Mycoplasma pneumoniae is made, and the patient responds to erythromycin therapy. Which of the following histologic changes is most likely responsible for the pulmonary symptoms in this patient?

A) Neutrophils within bronchioles, extending into alveoli.
B) Granulomas with Langhans giant cells.
C) Pulmonary infarcts with vascular occlusion by microorganisms.
D) Mononuclear interstitial infiltrate.
E) Collection of neutrophils and fibrin in the pleural space.

A

Mononuclear interstitial infiltrate

Mycoplasma infections lead to a primary atypical pneumonia in which there are no alveolar infiltrates, but there is prominent interstitial inflammation with lymphocytes, histiocytes, and plasma cells. Alveolar and bronchiolar neutrophilic exudates suggest a bacterial agent causing pneumonia. Granulomas with Langhans-type giant cells are typical of tuberculosis. Proliferation of microorganisms with vascular occlusion and infarction is most typical of Aspergillus fungal infections. An empyema with neutrophils suggests a bacterial cause for pneumonia with spread to pleura.

How well did you know this?
1
Not at all
2
3
4
5
Perfectly
29
Q

A study of transfusion-related infectious diseases determines that some blood donors appear to have acquired an infection via vertical transmission from mother to child. Laboratory testing strategies are devised to detect the most common of these infections and exclude such persons as blood donors. As a consequence, which of the following infectious agents is most likely to be a significant cause for rejection as a blood donor later in life?

Escherichia coli
Hepatitis B virus
Plasmodium vivax
Candida albicans
Pneumocystis carinii
A

HBV

Testing for hepatitis B and C is part of routine screening of blood donors. This form of transmission for hepatitis B is most common in developing nations. Escherichia coli can be a congenital infection, but it leaves no major significant lasting sequelae in infants who survive. Malaria, candidal infection, and pneumocystosis are not congenital infections.

How well did you know this?
1
Not at all
2
3
4
5
Perfectly
30
Q

A 6-year-old boy has had diarrhea for 7 days and is brought to the physician. He has averaged about six stools per day, which appear mucoid and sometimes blood-tinged. On physical examination, his temperature is 37.4°C. He has mild lower abdominal tenderness but no masses. A stool culture is positive for Shigella sonnei. Which of the following would most likely be seen in an endoscopic biopsy specimen from this child’s colon?

A) Epithelial disruption with overlying exudate of polymorphonuclear leukocytes (neutrophils).
B) Multiple granulomas throughout the colon wall
C) Slight increase in numbers of lymphocytes and plasma cells in lamina propria
D) Intranuclear inclusions in enterocytes
E) Extensive scarring of lamina propria with stricture formation

A

Epithelial disruption with overlying exudate of polymorphonuclear leukocytes (neutrophils).

Shigellosis results in bloody dysentery because the Shigella organisms can invade and destroy the mucosa. There is typically a mononuclear infiltrate extending to the lamina propria, with a neutrophilic exudate overlying the ulcerated areas. Granulomatous inflammation may be seen with granulomatous colitis (Crohn disease) and intestinal tuberculosis (rare). An increase in mononuclear inflammatory cells may be seen with milder forms of enterocolitis caused by viruses, Giardia, and Salmonella spp. Intranuclear inclusions in enterocytes point to infection with DNA viruses, such as herpesviruses. Stricture formation may follow intestinal tuberculosis.

How well did you know this?
1
Not at all
2
3
4
5
Perfectly
31
Q

A 3-year-old child has erosion of a roughened corneal surface caused by xerophthalmia. Keratomalacia results in corneal scarring with eventual blindness after several years. This ocular damage could most likely have been prevented by treating a dietary deficiency of which of the following nutrients?

Protein
Vitamin K
Iron
Niacin
Vitamin A
A

Vitamin A

Vitamin A is essential to maintain epithelia. The lack of vitamin A affects the function of lacrimal glands and conjunctival epithelium, promoting keratomalacia. Dietary protein is essential for building tissues, particularly muscle, but it has no specific effect in maintaining ocular structures. Vitamin K is beneficial for synthesis of coagulation factors by the liver to prevent bleeding problems. Iron is essential for production of heme, which is needed to manufacture hemoglobin in RBCs. Niacin is involved with nicotinamide in many metabolic pathways, and deficiency leads to diarrhea, dermatitis, and dementia.

How well did you know this?
1
Not at all
2
3
4
5
Perfectly
32
Q

A 5-year-old child is admitted to the hospital after ingesting pills he found in a cabinet at home. The child is rapidly becoming obtunded. Laboratory studies show a serum AST level of 850 U/L and ALT level of 1052 U/L. The child’s respiratory and cardiac status remain stable. Which of the following drugs was most likely ingested?

Acetaminophen
Penicillin
Aspirin
Sulfamethoxazole
Codeine
A

Acetaminophen.

Acetaminophen = paracetamol. Toxicity produces hepatic necrosis. This effect is enhanced with prior ethanol ingestion. Hepatic necrosis is indicated by extremely high levels of AST and ALT. Penicillin can cause systemic anaphylaxis in a few persons. Aspirin can produce a metabolic acidosis. Sulfa drugs may produce renal failure. Opiates, including codeine, are central nervous system and respiratory depressants.

How well did you know this?
1
Not at all
2
3
4
5
Perfectly
33
Q

A 23-year-old woman is delivered of a stillborn fetus at 36 weeks’ gestation. The woman experienced sudden onset of lower abdominal pain several hours before delivery. An abdominal ultrasound scan showed a large retroplacental hemorrhage. Delivery was accompanied by a 1200-mL blood loss. Maternal use of which of the following agents is most likely to be associated with these findings?

Acetaminophen
Amphetamine
Cannabinoids
Cocaine
Ethanol
Heroin
Lysergic acid
A

Cocaine

Cocaine has powerful vasoactive effects, including vasoconstriction. The effects on the placenta can include decreased blood flow with fetal hypoxia and spontaneous abortion, placentae abruption, and fetal hemorrhages. Chronic maternal cocaine use results in neurologic impairment of infants. Acetaminophen overdosage can produce hepatic necrosis and encephalopathy. Amphetamines may have acute cardiovascular effects, including arrhythmias. Cannabinoids act as mild tranquilizers and have no major tissue effects. Ethanol ingestion in pregnancy can be associated with fetal alcohol syndrome, which is not fatal but results in subtle anomalies and impaired development. Heroin ingestion can lead to fetal cardiorespiratory depression but not hemorrhage. Lysergic acid (LSD) use does not cause hemorrhage.

How well did you know this?
1
Not at all
2
3
4
5
Perfectly
34
Q

A 49-year-old man has had increasing knee and hip pain for the past 10 years. The pain is worse at the end of the day. During the past year, he has become increasingly drowsy at work. His wife complains that he is a “world class” snorer. During the past month, he has experienced bouts of sharp, colicky, right upper abdominal pain. On physical examination, his temperature is 37°C, pulse 82/min, respirations 10/min, and blood pressure 140/85 mm Hg. He is 175 cm (5 ft 8 in) tall and weighs 156 kg (BMI 51). Laboratory findings show glucose of 139 mg/dL, total cholesterol 229 mg/dL, and HDL cholesterol 33 mg/dL. An arterial blood gas measurement shows pH of 7.3, PCO2 50 mm Hg, and PO2 70 mm Hg. Which of the following conditions is most likely to be present?

Hashimoto thyroiditis
Hypertrophic cardiomyopathy
Laryngeal papillomatosis
Nonalcoholic steatohepatitis
Panlobular emphysema
Rheumatoid arthritis
A

NASH

This patient is morbidly obese with complications, including obesity hypoventilation syndrome, glucose intolerance, probable sleep apnea, cholelithiasis, and osteoarthritis. Macrovesicular steatosis with hepatomegaly is seen in obesity and may even progress to cirrhosis. Weight gain due to hypothyroidism, which could occur in Hashimoto thyroiditis, is modest and does not lead to morbid obesity. An “obesity cardiomyopathy” resembles dilated cardiomyopathy. Laryngeal papillomatosis, which produces airway obstruction (without snoring), occurs more often in children and is not associated with obesity. The blood gas findings in this case could be seen in emphysema, which is not a complication of obesity; panlobular emphysema is much less common than the centrilobular emphysema associated with smoking. Rheumatoid arthritis tends to involve small joints first, and there is no relationship to obesity

How well did you know this?
1
Not at all
2
3
4
5
Perfectly
35
Q

A 56-year-old man has had increasing lower leg swelling during the past 6 months. He also has had so much difficulty breathing at night that he sleeps propped up on two pillows. On physical examination, his temperature is 37.1°C, pulse 80/min, respirations 17/min, and blood pressure 110/70 mm Hg. On auscultation of the chest, bilateral crackles are audible at the lung bases. The liver span is increased. There is 2+ pitting edema to the thighs. Laboratory findings show hemoglobin of 13.4 g/dL, hematocrit 40.2%, MCV 88 μm3, platelet count 229,300/mm3, and WBC count 6715/mm3. One year later, he develops an acute psychosis. He dies of aspiration pneumonia. At autopsy, there is anterior vermian atrophy and petechial hemorrhages with brown discoloration in the periaqueductal gray matter, as well as shrunken mamillary bodies. A chronic deficiency of which of the following vitamins is most likely to explain these findings?

Vitamin A (retinoic acid)
Vitamin B1 (thiamine)
Vitamin B2 (riboflavin)
Vitamin B3 (niacin)
Vitamin B12 (cobalamin)
Vitamin C (ascorbic acid)
Vitamin D (cholecalciferol)
Vitamin E (α-tocopherol)
A

Thiamine

The patient has features of Wernicke disease and beriberi, both resulting from thiamine deficiency, which can accompany chronic alcoholism. Vitamin A deficiency leads to night blindness, keratomalacia, and respiratory tract difficulties due to epithelial disorders. Pure vitamin B2 deficiency is rare and is marked by findings such as neuropathy and cheilosis. Niacin deficiency leads to pellagra with dementia, diarrhea, and photodermatitis. Vitamin B12 deficiency leads to subacute combined degeneration of the spinal cord; the MCV in this case is not in the range to cause macrocytosis. Vitamin C deficiency leads to scurvy with anemia, loose teeth, hematomas, and poor wound healing. Vitamin D deficiency in an adult leads to osteomalacia with risk of fractures. Vitamin E deficiency is rare and produces spinal cord changes similar to those of vitamin B12 deficiency.

Adam notes: Mamillary bodies rely on thiamine for metabolism.

How well did you know this?
1
Not at all
2
3
4
5
Perfectly
36
Q

A 3-month-old previously healthy infant was found dead by his mother late one evening. When she put him in his crib 1 hour earlier, he showed no signs of distress. The infant’s term birth had followed an uncomplicated pregnancy, and he had been feeding well and gaining weight normally. Which of the following is the medical examiner most likely to find at autopsy?

Hyaline membrane disease
Cerebral cytomegalovirus
Tetralogy of Fallot
Adrenal neuroblastoma
No abnormalities
A

No abnormality.

The events described suggest sudden infant death syndrome (SIDS). The cause is unknown and, by definition, there are no significant gross or microscopic autopsy findings. Infants with congenital anomalies or infections are unlikely to appear healthy, feed well, or gain weight normally. Hyaline membrane disease (=RDS) occurs at birth with prematurity. Congenital neoplasms are a rare cause of sudden death.

How well did you know this?
1
Not at all
2
3
4
5
Perfectly
37
Q

A 19-year-old woman, G 2, P 1, has a screening ultrasound at 20 weeks’ gestation that shows no abnormalities. However, premature labor leads to an emergent vaginal delivery at 31 weeks. Soon after birth, the neonate develops respiratory distress requiring intubation with positive pressure ventilation. Which of the following prenatal diagnostic tests could have best predicted this neonate’s respiratory distress?

Maternal serum α-fetoprotein determination
Phospholipid analysis of amniotic fluid
Chromosomal analysis
Coombs test on cord blood
Genetic analysis for cystic fibrosis gene

A

Phospholipid analysis of amniotic fluid

The neonate most likely has hyaline membrane disease from fetal lung immaturity and lack of surfactant. Surfactant consists predominantly of dipalmitoyl phosphatidylcholine. The adequacy of surfactant production can be gauged by the phospholipid content of amniotic fluid, because fetal lung secretions are discharged into the amniotic fluid.

The maternal serum α-fetoprotein level is useful to predict fetal neural tube defects and chromosomal abnormalities. Chromosomal analysis may help to predict problems after birth or the possibility of fetal loss. The Coombs test may help to determine the presence of erythroblastosis fetalis. Cystic fibrosis does not cause respiratory problems at birth.

How well did you know this?
1
Not at all
2
3
4
5
Perfectly
38
Q

A 19-year-old primigravida who has had an uncomplicated pregnancy undergoes a screening ultrasound at 16 weeks’ gestation that shows no abnormalities. At 18 weeks, the woman develops a mild rash on her face. She gives birth to a stillborn severely hydropic male infant at 33 weeks. At autopsy, there are no congenital malformations, but cardiomegaly is present. The histologic appearance of the bone marrow shows erythroid precursors demonstrating large, pink, intranuclear inclusions. Which of the following is the most likely cause of these findings?

Maternal IgG crossing the placenta
Chromosomal anomaly of the fetus
Congenital neuroblastoma
Inheritance of two abnormal CFTR genes
Infection with parvovirus B19
A

Infection with parvovirus B19

The erythroid precursors demonstrate large, pink, intranuclear inclusions typical of parvovirus. In adults, such an infection typically causes fifth disease, which is self-limiting. However, this is one of the “O” infections in the TORCH mnemonic describing congenital infections (toxoplasmosis, other infections, rubella, cytomegalovirus infection, and herpes simplex infection). Parvovirus infection in the fetus can lead to a profound fetal anemia with cardiac failure and hydrops fetalis. Erythroblastosis fetalis is unlikely to occur in a first pregnancy, and only erythroid expansion is present, not erythroid inclusions. Although a variety of chromosomal anomalies—monosomy X, in particular—may lead to hydrops, malformations are typical. Congenital tumors are an uncommon cause of hydrops, and they would produce a mass lesion, which was not described in this case. Cystic fibrosis does not affect erythropoiesis.

How well did you know this?
1
Not at all
2
3
4
5
Perfectly
39
Q

An infant born at term develops abdominal distention in the first week of life. Meconium ileus is diagnosed. The infant has persistent steatorrhea and fails to develop normally. Later in childhood, multiple respiratory tract infections lead to widespread bronchiectasis. Which of the following laboratory findings is most likely related to this child’s underlying disease?

Decreased serum thyroxine level
Positive HIV serology
Elevated sweat chloride level
Increased urine homovanillic acid level
Hyperbilirubinemia
A

Elevated sweat chloride level.

The findings are typical of cystic fibrosis, which is an inherited defect in chloride transport.

Cretinism from hypothyroidism results in impaired central nervous system and skeletal development. An infant with congenital HIV infection may have a variety of opportunistic infections but not meconium ileus. Increased urine homovanillic acid is a feature of neuroblastoma, a mass lesion that could also cause bowel obstruction but not meconium ileus. Neonatal jaundice has a variety of causes, including the inherited disorder galactosemia.

How well did you know this?
1
Not at all
2
3
4
5
Perfectly
40
Q

An 80-year-old man with a lengthy history of smoking survived a small myocardial infarction several years ago. He now reports chest and leg pain during exercise. On physical examination, his vital signs include temperature of 36.9°C, pulse 81/min, respirations 15/min, and blood pressure 165/100 mm Hg. Peripheral pulses are poor in the lower extremities. There is a 7-cm pulsating mass in the midline of the lower abdomen. Laboratory studies include two fasting serum glucose measurements of 170 mg/dL and 200 mg/dL. Which of the following vascular lesions is most likely to be present in this patient?

Aortic dissection
Arteriovenous fistula
Atherosclerotic aneurysm
Glomus tumor
Polyarteritis nodosa
Takayasu arteritis
Thromboangiitis obliterans
A

AAA

Abdominal aneurysms are most often related to underlying atherosclerosis. This patient has multiple risk factors for atherosclerosis, including diabetes mellitus, hypertension, and smoking. When the aneurysm reaches this size, there is a significant risk of rupture.

An aortic dissection is typically a sudden, life-threatening event with dissection of blood out of the aortic lumen, typically into the chest, without a pulsatile mass. The risk factors of atherosclerosis and hypertension underlie aortic dissection. An arteriovenous fistula can produce an audible bruit on auscultation. Glomus tumors are usually small peripheral masses. Polyarteritis nodosa can produce small microaneurysms in small arteries. Takayasu arteritis typically involves the aortic arch and branches in children. Thromboangiitis obliterans (Buerger disease) is a rare condition with occlusion of the muscular arteries of the lower extremities in smokers.

How well did you know this?
1
Not at all
2
3
4
5
Perfectly
41
Q

In the third match of a volleyball tournament, a 15-year-old girl jumps up for a block and collapses. Despite cardiopulmonary resuscitation, she cannot be revived. She had been healthy all her life and complained only of limited episodes of chest pain in games during the current school year. Which of the following pathologic findings of the heart is the medical examiner most likely to find?

A) Haphazardly arranged hypertrophied septal myocytes
B) Extensive myocardial hemosiderin deposition
C) Tachyzoites within foci of myocardial necrosis and inflammation
D) Mitral valvular stenosis with left atrial enlargement
E) Large, friable vegetations with destruction of aortic valve cusps

A

A

Hypertrophic cardiomyopathy is the most common cause of sudden unexplained death in young athletes. There is asymmetric septal hypertrophy that reduces the ejection fraction of the left ventricle, particularly during exercise. Histologically, haphazardly arranged hypertrophic myocardial fibers are seen.

Hemochromatosis gives rise to a restrictive cardiomyopathy in middle age. Tachyzoites of Toxoplasma gondii signify myocarditis, a process that may occur in immunocompromised persons. Rheumatic heart disease with chronic valvular changes would be unusual in a patient this age, and the course is most often slowly progressive. Valve destruction with vegetations is seen in infective endocarditis. This would be accompanied by signs of sepsis

42
Q

A 77-year-old woman sees her physician for a routine health maintenance examination. On physical examination, she is afebrile. Her pulse is 66/min, respirations 14/min, and blood pressure 125/85 mm Hg. On auscultation, a systolic ejection murmur is heard. There are a few crackles over the lung bases posteriorly. From the representative gross appearance of the aortic valve showing degenerative change, which of the following most likely contributed to the development of this lesion?

Chromosomal aneuploidy
Aging
Tertiary syphilis
Atherosclerosis
Systemic lupus erythematosus
A

Aging

This patient has calcific aortic stenosis of a valve with three cusps, a degenerative change that may occur in a normal aortic valve with aging. Congenital anomalies with chromosomal aneuploidies (e.g., trisomy 21) are unlikely to be associated with aortic stenosis or a bicuspid valve. In syphilis, the aortic root dilates and aortic insufficiency results. Atherosclerosis does not produce valvular disease from involvement of the valve itself. Systemic lupus erythematosus may give rise to small sterile vegetations on mitral or tricuspid valves, but these rarely cause valve disease.

43
Q

A 19-year-old man suddenly collapses and is brought to the emergency department. His vital signs include temperature of 37.1°C, pulse 84/min, respirations 18/min, and blood pressure 80/40 mm Hg. Laboratory findings include hemoglobin of 13.5 g/dL, platelet count 252,000/mm3, WBC count 7230/mm3, serum glucose 73 mg/dL, and creatinine 1.2 mg/dL. The total creatine kinase (CK) level is elevated, with a CK-MB fraction of 10%. Which of the following underlying conditions is most likely to be present in this patient?

Hereditary hemochromatosis
Marfan syndrome
Down syndrome
DiGeorge syndrome
Familial hypercholesterolemia
A

Familial hypercholesterolaemia.

The laboratory findings suggest an acute myocardial infarction. Persons with familial hypercholesterolemia have accelerated and advanced atherosclerosis, even by the second or third decade.

Hereditary hemochromatosis may result in an infiltrative cardiomyopathy with iron overload, more typically in the fifth decade. Marfan syndrome may result in aortic dissection or floppy mitral valve. DiGeorge syndrome can be associated with a variety of congenital heart defects, but survival with this syndrome is usually limited by infections due to the cell-mediated immunodeficiency.

44
Q

A 27-year-old woman gives birth to a term infant after an uncomplicated pregnancy and delivery. The infant is cyanotic at birth. Two months later, physical examination shows the infant to be at the 37th percentile for height and weight. Which of the following is the most likely diagnosis?

Tetralogy of Fallot
Pulmonic stenosis
Truncus arteriosus
Transposition of the great vessels
Aortic stenosis
A

Transposition of the great vessels.

The aorta emerges from the right ventricle, and the pulmonic trunk exits the left ventricle. Unless there is another anomalous connection between the pulmonary and systemic circulations, this condition is not compatible with extrauterine life. The most common anomalous connections would be ventricular septal defect (VSD), patent ductus arteriosus, and patent foramen ovale (or atrial septal defect). In tetralogy of Fallot, the aorta overrides a VSD but is not transposed. In pulmonic and aortic stenosis, the great arteries are normally positioned but small. In truncus arteriosus, the spiral septum that embryologically separates the great arteries does not develop properly.

45
Q

A 69-year-old woman complains of increasing back pain for 1 month. On physical examination, there is tenderness over the lower back but no kyphosis or scoliosis. A radiograph of the spine shows a partial collapse of T11 and several 0.5- to 1.5-cm lytic lesions with a rounded “soap-bubble” appearance in the thoracic and lumbar vertebrae. A bone marrow biopsy is performed, and a smear of the aspirate is shown. Which of the following is the most likely laboratory finding in this patient?

Monoclonal gammopathy in the serum
t(9;22) in the karyotype of marrow
Elevated leukocyte alkaline phosphatase score
Decreased serum alkaline phosphatase level
Platelet count of 750,000/mm3
WBC count of 394,000/mm3

A

Monoclonal gammopathy in the serum

The characteristic “punched-out” bone lesions of multiple myeloma seen on radiographs result from areas of bone lysis and plasma cell proliferation. The bone marrow aspirate shows plasma cells. The monoclonal population of plasma cells often produces a monoclonal serum “spike” seen in serum or urine protein electrophoresis. Patients can have hypercalcemia and an increased serum alkaline phosphatase level. The neoplastic cells are generally well differentiated, with features such as a perinuclear hof, similar to normal plasma cells.

The t(9;22) translocation is the Philadelphia chromosome seen in chronic myelogenous leukemia (CML). CML and other myeloproliferative disorders are sometimes accompanied by a thrombocytosis but are unlikely to produce mass lesions or bony destruction. Leukemias also can fill the marrow space but generally do not destroy bone.

46
Q

A 39-year-old man experiences sudden onset of a severe headache. Physical examination shows no localizing neurologic signs and no organomegaly. A stool sample is positive for occult blood. Areas of purpura appear on the skin of his extremities. Laboratory studies show hemoglobin of 9.6 g/dL, hematocrit 28.9%, platelet count 26,400/mm3, and WBC count 75,000/mm3. The peripheral blood smear has the appearance shown, and schistocytes are also seen. The plasma D dimer level (fibrin degradation products), prothrombin time, and partial thromboplastin time are all elevated. Cytogenetic analysis of cells from a bone marrow biopsy specimen is most likely to yield which of the following karyotypic abnormalities?

t(8;21)
t(9;22)
t(14;18)
t(15;17)
t(8;14)
A

t(15;17)

This peripheral blood smear is characteristic of acute promyelocytic leukemia (M3 class of acute myelogenous leukemia), with many promyelocytes containing prominent azurophilic granules and short, red, cytoplasmic, rodlike inclusions called Auer rods. Release of the granules can trigger the coagulation cascade, leading to disseminated intravascular coagulation (DIC). As in this case, many patients develop DIC. The t(15;17) translocation is characteristic of this disease; it results in the fusion of the retinoic acid receptor gene on chromosome 17 with the promyelocytic leukemia gene on chromosome 15. The fusion gene results in elaboration of an abnormal retinoic acid receptor that blocks myeloid differentiation. Therapy with all-trans retinoic acid (vitamin A) can alleviate the block and induce remission in many patients.

The t(8:21) abnormality is seen in the M2 variant of acute myelogenous leukemia. The t(9:22) translocation gives rise to the Philadelphia chromosome of chronic myelogenous leukemia. A t(14:18) karyotype suggests a follicular lymphoma. The t(8:14) translocation can be seen in patients with Burkitt lymphoma.

47
Q

A 60-year-old man has experienced vague abdominal discomfort accompanied by bloating and diarrhea for the past 6 months. On physical examination, there is a midabdominal firm mass. The stool is positive for occult blood. An abdominal CT scan shows a 5 × 12 cm mass involving the wall of the distal ileum and adjacent mesentery. A laparotomy is performed, and the mass is removed. Microscopically, the mass is composed of sheets of large lymphoid cells with large nuclei, prominent nucleoli, and frequent mitoses. The neoplastic cells mark with CD19+ and CD20+ and have the BCL6 gene rearrangement. Which of the following prognostic features is most applicable to this case?

A) Indolent disease with survival of 7 to 9 years without treatment
B) Aggressive disease that can be cured by aggressive chemotherapy
C) Aggressive disease that does not respond to chemotherapy and transforms to acute leukaemia
D) Indolent disease that can be cured by chemotherapy
E) Indolent disease that often undergoes spontaneous remission

A

Aggressive disease that can be cured by aggressive chemo

This patient has the clinical and morphologic features of diffuse large cell lymphoma of B cells. These tumors often involve extranodal sites, show large anaplastic lymphoid cells that involve the tissues diffusely, and contain BCL6 gene rearrangements. Their clinical course is aggressive, and they become rapidly fatal if untreated. However, with intensive chemotherapy, 60% to 80% of patients achieve complete remission, and about 50% can be cured.

Adam notes - DLBCL can be formed from Richter’s transformation of CLL.

48
Q

A 60-year-old woman has had headaches and dizziness for the past 5 weeks. She has been taking cimetidine for heartburn and omeprazole for ulcers. On physical examination, she is afebrile and normotensive, and her face has a plethoric to cyanotic appearance. There is mild splenomegaly but no other abnormal findings. Laboratory studies show hemoglobin of 21.7 gm/dL, hematocrit 65%, platelet count 400,000/mm3, and WBC count 30,000/mm3 with 85% polymorphonuclear leukocytes, 10% lymphocytes, and 5% monocytes. The peripheral blood smear shows abnormally large platelets. The serum erythropoietin level is undetectable, but the ferritin level is normal. Which of the following is most characteristic of the natural history of this patient’s disease?

A) Death from transformation of the disease into acute B-lymphoblastic leukemia
B) Onset of marrow fibrosis with extensive extramedullary hematopoiesis in the spleen
C) Spontaneous remissions and relapses without any treatment
D) Gradual increase in monoclonal serum immunoglobulins
E) Development of a gastric lymphoma

A

B

This patient has polycythemia vera. The symptoms result from the increased hematocrit and blood volume. Undetectable erythropoietin in the face of polycythemia is characteristic of polycythemia vera. Polycythemia vera is a myeloproliferative disorder in which the neoplastic myeloid cells differentiate preferentially along the erythroid lineage. However, other lineages also are affected; hence, there is leukocytosis and thrombocytosis. These patients are Ph1-chromosome negative. Untreated, these patients die of episodes of bleeding or thrombosis—both related to disordered platelet function and the hemodynamic effects on distended blood vessels. Treatment by phlebotomy reduces the hematocrit. With this treatment, the disease in 15% to 20% of patients characteristically transforms into myelofibrosis with myeloid metaplasia. Termination in acute leukemia, unlike in chronic myeloid leukemia, is rare. When it occurs, it is an acute myeloid leukemia, not lymphoblastic leukemia.

49
Q

A 30-year-old previously healthy man has had an enlarging nodular area on his arm for the past 8 months. On physical examination, there is an ulcerated, reddish-violet, 3 x 7 cm lesion on his right forearm and nontender right axillary and left inguinal lymphadenopathy. A chest radiograph shows a 4-cm nodular left pleural mass. An abdominal CT scan shows a 5-cm right retroperitoneal mass. Biopsy of an inguinal node is performed, and microscopic examination shows large anaplastic cells, some of which contain horseshoe-shaped nuclei and voluminous cytoplasm. The tumor cells cluster around venules and infiltrate sinuses. He goes into remission following chemotherapy. Which of the following immunohistochemical markers is most likely to be positive in the tumor cells?

Anaplastic lymphoma kinase protein
CD4
cKIT proto-oncogene
HTLV-1
p24 antigen
A

Anaplastic lymphoma kinase protein

This patient has a form of T-cell neoplasm known as anaplastic large cell lymphoma (ALCL), which most often appears in children and young adults. It is often extranodal and has a characteristic gene rearrangement on chromosome 2p23 that results in production of anaplastic lymphoma kinase (ALK) with tyrosine kinase activity.

CD4 can be a marker for adult T-cell leukemia/lymphoma, which results from infection by the retrovirus HTLV-1. The T-cell proliferations involving skin, known as mycosis fungoides/Sézary syndrome, are also CD4 positive. The cKIT proto-oncogene has been associated with some NK cell lymphomas. The p24 antigen is part of HIV, which is most often associated with B-cell neoplasms.

50
Q

A 32-year-old woman visits her physician because she has experienced fatigue, fever, night sweats, and painless lumps in the right side of her neck for the past 3 months. On physical examination, her temperature is 37.5°C. She has right cervical nontender lymphadenopathy. One of the lymph nodes is biopsied, and the histologic finding is shown at high power to have cells with a bilobed nucleus with prominent eosinophilic inclusion-like nucleoli. Which of the following is the most likely diagnosis?

Burkitt lymphoma
Well-differentiated lymphocytic lymphoma
Reactive lymphoid hyperplasia
Hodgkin disease, nodular sclerosis type
Cytomegalovirus infection
A

Hodgkin disease

The figure shows a Reed-Sternberg cell, which is characteristic of Hodgkin disease. Notice also the nonneoplastic eosinophils, lymphocytes, and macrophages in the background. Reed-Sternberg cells are the neoplastic component in Hodgkin disease, and unlike non-Hodgkin lymphomas, these transformed cells are scattered in a background of a proliferation of nonneoplastic inflammatory cells.

As a historical note, the current Rye classification of Hodgkin disease grew out of an international conference held in Rye, Scotland. Nothing much happened at the conference, because everyone wanted to play golf. The meeting’s organizer realized that little progress was being made, and he assembled a few of his friends at the hotel bar the night before the conference was to end. After a few whiskeys, he got them to agree on a simple scheme with four categories, and it has not been revised since—unlike the many classification schemes for non-Hodgkin lymphomas.

51
Q

A 71-year-old woman has had a history of back pain for several months. She recently developed a cough productive of yellowish sputum, and a sputum culture grew Streptococcus pneumoniae. On physical examination, she has no lymphadenopathy or hepatosplenomegaly. Laboratory findings show Na+ of 141 mmol/L, K+ 4.2 mmol/L, Cl− 104 mmol/L, CO2 26 mmol/L, glucose 79 mg/dL, calcium 9.8 mg/dL, phosphorus 3.1 mg/dL, AST 31 U/L, ALT 24 U/L, alkaline phosphatase 291 U/L, total protein 9.3 g/dL, albumin 4.4 g/dL, urea nitrogen 31 mg/dL, and creatinine 3.8 mg/dL. A skull radiograph shows multiple 1- to 4-cm lytic lesions. Which of the following laboratory findings is most likely to be reported?

Bence-Jones proteinuria
Increased hemoglobin F
Hemoglobin 21.2 g/dL
Positive ANA
WBC count 450,000/mm3
A

Bence Jones proteinuria

The findings are those of multiple myeloma with increased M protein and lytic bone lesions producing bone pain; there is increased risk of infection from encapsulated bacteria. Elevated hemoglobin F occurs in some hemoglobinopathies such as β-thalassemias, which do not have hypergammaglobulinemia. Polycythemia can be a feature of myeloproliferative disorders but not myeloma. The ANA test result is positive in autoimmune conditions but not in lymphoproliferative disorders. A markedly elevated WBC count suggests leukemia, typically chronic myelogenous leukemia

52
Q

A 60-year-old man has developed widespread ecchymoses over the skin in the past month. His medical history includes a diagnosis of mucinous adenocarcinoma of the rectum. On physical examination, he appears cachectic and pale. An abdominal CT scan shows multiple hepatic masses. Laboratory studies show prothrombin time of 30 sec, partial thromboplastin time of 55 sec, platelet count 15,200/mm3, fibrinogen level 75 mg/dL, and fibrin split product levels (D dimer) that are very elevated. Which of the following morphologic findings is most likely to be present on examination of his peripheral blood smear?

Howell-Jolly bodies
Teardrop cells
Macro-ovalocytes
Schistocytes
Target cells
A

Schistocytes

This is an example of disseminated intravascular coagulation (DIC) with associated microangiopathic hemolytic anemia. The DIC developed in the setting of a mucin-secreting adenocarcinoma.

Howell-Jolly bodies are small, round inclusions in RBCs that appear when the spleen is absent. Teardrop cells are most characteristic of myelofibrosis and other infiltrative disorders of the marrow. Macro-ovalocytes are seen in megaloblastic anemias, such as vitamin B12 deficiency. Target cells appear in hemoglobin C disease or severe liver disease.

53
Q

A 23-year-old woman in her 25th week of pregnancy has felt no fetal movement for the past 3 days. Three weeks later, she still has not given birth and suddenly develops dyspnea with cyanosis. On physical, examination, her temperature is 36.9°C, pulse 102/min, respirations 21/min, and blood pressure 80/40 mm Hg. She has large ecchymoses over the skin of her entire body. A stool sample is positive for occult blood. Laboratory studies show an elevated prothrombin time and partial thromboplastin time. The platelet count is decreased, plasma fibrinogen is markedly decreased, and fibrin split products are detected. Which of the following is the most likely cause of the bleeding diathesis?

Increased vascular fragility
Toxic injury to the endothelium
Reduced production of platelets
Increased consumption of clotting factors and platelets
Defects in platelet adhesion and aggregation

A

Increased consumption of clotting factors and platelets

The presence of thrombocytopenia, increased prothrombin and partial thromboplastin values, fibrin split products, and the low fibrinogen concentration all suggest disseminated intravascular coagulation (DIC), which was most likely caused by a retained dead fetus, an obstetric complication that can lead to DIC through release of thromboplastins from the fetus. This release causes widespread microvascular thrombosis and consumes clotting factors and platelets. There is no damage to the vascular endothelium or vascular wall. Platelet production is normal, but platelets are consumed by widespread thrombosis of small vessels. There is no defect in platelet function.

54
Q

A clinical study of patients who inherit mutations that reduce the level of spectrin in the RBC membrane cytoskeleton shows an increased prevalence of chronic anemia with splenomegaly. For many of these patients, it is observed that splenectomy reduces the severity of anemia. This beneficial effect of splenectomy is most likely related to which of the following processes?

Increase in synthesis of spectrin in RBCs
Increase in deformability of RBCs
Decrease in opsonization of RBCs
Decrease in trapping of RBCs in the spleen
Decrease in production of reactive oxygen species

A

Decrease in trapping of RBCs in the spleen

In patients with hereditary spherocytosis, spheroidal cells are trapped and destroyed in the spleen because the abnormal RBCs have reduced deformability. Splenectomy is beneficial because the spherocytes are no longer detained by the spleen. Splenectomy has no effect on the synthesis of spectrin or RBC deformability as a result; the RBCs in spherocytosis are not killed by opsonization. In warm antibody hemolytic anemias, opsonized RBCs are removed by the spleen. Reactive oxygen species do not play a role in anemias.

55
Q

A 10-year-old child has experienced multiple episodes of pneumonia and meningitis with septicemia since infancy. Causative organisms that have been cultured include Streptococcus pneumoniae and Haemophilus influenzae. On physical examination, the child has no organomegaly and no deformities. Laboratory studies show hemoglobin of 9.2 g/dL, hematocrit 27.8%, platelet count 372,000/mm3, and WBC count 10,300/mm3. A hemoglobin electrophoresis shows 1% hemoglobin A2, 7% hemoglobin F, and 92% hemoglobin S. Which of the following is the most likely cause of the repeated infections in this child?

Loss of normal splenic function from recurrent ischemic injury
Reduced synthesis of immunoglobulins
Impaired neutrophil production
Reduced synthesis of complement proteins by the liver
Reduced expression of adhesion molecules on endothelial cells

A

Loss of normal splenic function from recurrent ischaemic injury

In sickle cell anemia, the cumulative damage to the spleen results in autosplenectomy, leaving behind a small fibrotic remnant of this organ. The impaired splenic function and resultant inability to clear bacteria from the bloodstream can occur early in childhood, leading to infection with encapsulated bacterial organisms. Thus, immunodeficiency results from lack of splenic function, not from lack of immunoglobulins. There is no impairment in production or function of neutrophils. Levels of serum complement are normal. Adhesion between endothelial cells and RBCs is increased in sickle cell anemia.

56
Q

A 17-year-old girl has had frequent nosebleeds since childhood. Her gums bleed easily, even with routine tooth brushing. She has experienced menorrhagia since menarche at age 14. On physical examination, there are no abnormal findings. Laboratory studies show hemoglobin 14.1 g/dL, hematocrit 42.5%, MCV 90 μm3, platelet count 277,400/mm3, and WBC count 5920/mm3. Her platelets fail to aggregate in response to ADP, collagen, epinephrine, and thrombin. The ristocetin agglutination test result is normal. There is a deficiency of glycoprotein IIb-IIIa. Her prothrombin time is 12 sec and her partial thromboplastin time is 28 sec. Which of the following is the most likely diagnosis?

Disseminated intravascular coagulation
Glanzmann thrombasthenia
Immune thrombocytopenic purpura
Vitamin C deficiency
Von Willebrand disease
A

Glanzmann thrombasthenia

It is a rare autosomal recessive disorder with defective platelet aggregation from deficiency or dysfunction of glycoprotein IIb-IIIa. Disseminated intravascular coagulation results in consumption of all coagulation factors as well as platelets, so the prothrombin time and partial thromboplastin time are elevated, with thrombocytopenia. Immune thrombocytopenic purpura is caused by antibodies to platelet membrane glycoproteins IIb-IIIa or Ib-IX.

Scurvy due to vitamin C deficiency causes bleeding into soft tissues and skin from increased capillary fragility, but platelet number and function are normal. Von Willebrand disease is one of the most common bleeding disorders and results from qualitative or quantitative defects in vWF.

57
Q

A 9-year-old boy has developed prominent bruises on his extremities over the past week. On physical examination, he has ecchymoses and petechiae on his arms and legs. Laboratory studies show hemoglobin 13.8 g/dL, hematocrit 41.9%, MCV 93 μm3, platelet count 22,300/mm3, and WBC count 7720/mm3. He had respiratory syncytial virus pneumonia 3 weeks ago. His condition improves with corticosteroid therapy. Which of the following abnormalities is most likely to cause his hemorrhagic diathesis?

Anti-platelet antibodies
Bone marrow aplasia
Glycoprotein IIb-IIIa dysfunction
Vitamin C deficiency
Von Willebrand factor metalloprotease deficiency
A

Anti-platelet antibodies

Both acute immune thrombocytopenic purpura and immune thrombocytopenic purpura (ITP) are caused by antiplatelet autoantibodies, but the acute form is typically seen in children following a viral disease. If the bone marrow were aplastic, then all cell lines should be reduced.

Glycoprotein IIb-IIIa dysfunction/deficiency can be seen with Glanzmann thrombasthenia and chronic ITP. Scurvy due to vitamin C deficiency leads to increased capillary fragility with ecchymoses but not to thrombocytopenia. VWF metalloprotease (ADAMTS13) deficiency is a feature of thrombotic thrombocytopenic purpura (TTP)

58
Q

A 33-year-old woman has had increasing dyspnea with cough for the past 10 days. Over the past 2 days, her cough has become productive of chunks of gelatinous sputum. On physical examination, she is afebrile. There is extensive dullness to percussion over all lung fields. A chest radiograph shows diffuse opacification bilaterally. A transbronchial biopsy is performed, and light microscopic examination has the appearance showing the distal air spaces are filled with a granular, eosinophilic material that is positive with the PAS stain . On electron microscopy, there are many lamellar bodies. Antibody directed against which of the following substances is most likely to cause her illness?

α1-antitrypsin
CFTR
DNA topoisomerase I
Glomerular basement membrane
Granulocyte macrophage colony stimulating factor
Neutrophilic myeloperoxidase
A

GM-CSF

The patient has the acquired form of pulmonary alveolar proteinosis (PAP), an uncommon condition of unknown etiology. Ten percent of PAP cases are congenital due to mutations in the granulocyte macrophage colony stimulating factor gene. Both forms of PAP have impaired surfactant clearance by alveolar macrophages.

α1-antitrypsin deficiency leads to panlobular emphysema. CFTR gene mutations lead to cystic fibrosis and widespread bronchiectasis. Anti-DNA topoisomerase I antibodies are seen in diffuse scleroderma, which produces interstitial fibrosis. Anti–glomerular basement membrane antibody is present in Goodpasture syndrome with extensive alveolar hemorrhage. Neutrophilic myeloperoxidase is a form of anti–neutrophil cytoplasmic autoantibody seen in Wegener granulomatosis.

59
Q

A local epidemic occurs among children at a summer camp, and they all develop upper respiratory tract infections manifested by coryza, pharyngitis, and tracheobronchitis. The children have fever and malaise but minimal sputum production. The total leukocyte count is not markedly elevated. Mycoplasma pneumoniae is cultured from the nasopharynx of many of the children. Which of the following histologic patterns is most likely to be found in the lungs of these children to explain these findings?

Alveolar neutrophilic exudates
Perivascular granulomatous inflammation
Hemorrhagic infarction
Hyaline membrane formation
Interstitial mononuclear cell infiltrates
A

Interstitial mononuclear cell infiltrates

Primary atypical pneumonias may result from a variety of infectious agents, including viruses, chlamydiae, and rickettsiae, although mycoplasmal infections are most common in children and young adults. About half of cases of Mycoplasma infection are accompanied by an increased cold agglutinin titer.

Neutrophilic exudates are typical of bacterial pneumonias. Granulomatous inflammation can appear with vasculitides and infections such as tuberculosis. A hemorrhagic infarction is most often the result of a pulmonary thromboembolus, although infection with Aspergillus may lead to vascular invasion and thrombosis. Hyaline membranes are seen in acute lung injuries (diffuse alveolar damage).

60
Q

A 46-year-old woman goes to the physician for a routine health maintenance examination. On physical examination, there are no remarkable findings. Her BMI is 22. She does not smoke. A tuberculin skin test is positive. A chest radiograph shows a solitary, 3-cm left upper lobe mass. The mass is removed at thoracotomy by wedge resection. The microscopic appearance of this lesion is depicted. The figure shows pink, amorphous tissue at the lower left, representing caseous necrosis. Which of the following is the most likely diagnosis?

Pulmonary hamartoma
Pulmonary infarction
Mycobacterium tuberculosis infection
Lung abscess
Primary adenocarcinoma
A

TB

The rim of the granuloma has epithelioid cells and Langhans giant cells. Caseating granulomatous inflammation is most typical of Mycoplasma tuberculosis infection. A hamartoma is a benign neoplastic process, and the mass is composed of pulmonary tissue elements, including cartilage and bronchial epithelium. A pulmonary infarct should have extensive hemorrhage. A lung abscess would have an area of liquefactive necrosis filled with tissue debris and neutrophils. A carcinoma may have central necrosis, not caseation, and there would be atypical, pleomorphic cells forming the mass.

61
Q

A 37-year-old woman comes to her physician because she has had a chronic nonproductive cough for 4 months. During this time, she has experienced loss of appetite and a 6-kg weight loss. She does not smoke. She is employed by a religious order. On physical examination, she is afebrile, and there are no remarkable findings. The chest radiograph shows a right peripheral subpleural mass. A fine-needle aspiration biopsy is performed, and the patient undergoes a right lower lobectomy. She remains free of symptoms for the next 10 years. Which of the following neoplasms did she most likely have?

Adenocarcinoma
Bronchial carcinoid
Bronchioloalveolar carcinoma
Hamartoma
Large cell carcinoma
Localized mesothelioma
Metastatic follicular carcinoma
A

Adenocarcinoma

The most common primary lung malignancy in women and in nonsmokers is adenocarcinoma. Overall, lung cancers in nonsmokers are far less frequent than in smokers. Primary adenocarcinomas in the lung tend to be small, peripheral masses that are amenable to surgical excision and have a better overall prognosis than other forms of lung cancer.

Bronchial carcinoids are uncommon endobronchial lesions. Bronchioloalveolar carcinomas are peripheral masses with a distinctive microscopic appearance of neoplastic cells proliferating along the alveolar and bronchiolar framework. Hamartomas are small, peripheral masses that contain benign epithelial and connective tissue elements. Large cell carcinomas are too poorly differentiated to be called adenocarcinomas or squamous cell carcinomas. Localized mesotheliomas are not related to asbestos exposure and are pedunculated masses attached to the pleura.

Overall, far more metastatic adenocarcinomas involve the lung than do primary adenocarcinomas (such as thyroid follicular carcinoma), but metastases tend to be multiple, and because they denote systemic disease, surgery generally is not an option.

62
Q

A 62-year-old woman who has had a chronic cough for years becomes short of breath after climbing a single flight of stairs. A chest radiograph performed 1 year ago showed increased lucency of upper lung fields and bilateral flattening of the diaphragmatic leaves. She has had nausea and vague abdominal discomfort for 6 months. Biopsy specimens from an upper gastrointestinal endoscopic study show a chronic nonspecific gastritis with no detectable Helicobacter pylori organisms. During the past month, she has passed red-colored urine on several occasions. Cystoscopic examination shows a 3-cm exophytic mass in the dome of the bladder, and biopsy specimens show a transitional cell carcinoma. Which of the following is most likely to lead to this spectrum of findings?

α1-Antitrypsin deficiency
Chronic alcoholism
Cigarette smoking
Exposure to aniline dye
Vitamin C deficiency
A

Smoking

The patient has findings that are consistent with emphysema. Smoking promotes gastritis. Several malignancies are related to smoking, including urinary tract transitional cell carcinoma and renal cell carcinoma. α1-Antitrypsin (AAT) deficiency can explain emphysema, but it would be panlobular, and AAT deficiency is not associated with urinary tract neoplasia. Chronic alcoholism can explain gastritis but not emphysema or carcinoma. Aniline dyes increase the risk of transitional cell carcinoma but not of emphysema or gastritis. Vitamin C deficiency can lead to soft tissue hemorrhages and bone pain but not to carcinoma or emphysema.

63
Q

For the past 4 months, a 50-year-old man has had difficulty breathing through his nose and also has experienced dull facial pain. On physical examination, there is a mass filling the right nasal cavity. CT scan of the head shows a 4-cm mass in the nasopharynx on the right that erodes adjacent bone. The mass is excised, and microscopic examination shows that it is composed of large epithelial cells with indistinct borders and prominent nuclei. Mature lymphocytes are scattered throughout the undifferentiated neoplasm. Which of the following etiologic factors most likely played the greatest role in the development of this lesion?

Epstein-Barr virus infection
Sjögren syndrome
Smoking tobacco
Allergic rhinitis
Wegener granulomatosis
A

EBV

This patient has a nasopharyngeal carcinoma. There is a strong association with Epstein-Barr virus infection, which contributes to the transformation of squamous epithelial cells.

Sjögren syndrome is associated with malignant lymphomas, but these typically arise in the salivary gland, not the nasal cavity. Smoking is not associated with nasopharyngeal carcinoma, although it does contribute to oral and esophageal cancers. Allergic rhinitis is associated with development of nasal polyps, but these do not become malignant. Wegener granulomatosis can involve the respiratory tract, causing granulomatous inflammation and vasculitis, but the nasopharyngeal region is not commonly affected, and there is no risk of malignant growth.

64
Q

A 62-year-old man sees his physician because he has had fever and back pain for the past 2 days. Physical examination shows tenderness of the right costovertebral angle. Laboratory studies show leukocytosis and pyuria with WBC casts. He has been receiving antibiotic therapy with cefotaxime, clindamycin, and nafcillin for the past 16 days. He now develops lower abdominal pain and a severe diarrhea. Clostridium difficile toxin is identified in a stool specimen. Which of the following conditions is he most likely to have now developed?

Appendicitis
Collagenous colitis
Diverticulitis
Ischemic colitis
Pseudomembranous colitis
Spontaneous bacterial peritonitis
Typhlitis
A

Pseudomembranous colitis

Pseudomembranous colitis, caused by overgrowth of Clostridium difficile, occurs when the normal gut flora is altered by broad-spectrum antibiotic therapy. In this case, the patient was treated for acute pyelonephritis. Organisms other than C. difficile, such as Candida, may overgrow, but C. difficile is the most common organism identified with pseudomembranous colitis.

The other choices are not related to antibiotic therapy. Appendicitis has a peak incidence at a younger age, and pain is more often localized in the right lower quadrant. Collagenous colitis is not common; it most often leads to watery diarrhea in middle-aged women. Diverticulitis can produce lower abdominal pain. Ischemic colitis may produce infarction with rupture and peritonitis. Spontaneous bacterial peritonitis occurs in the setting of ascites. Typhlitis is a rare condition with cecal inflammation; it occurs in immunocompromised persons.

65
Q

A 27-year-old man has sudden onset of marked abdominal pain. On physical examination, his abdomen is diffusely tender and distended, and bowel sounds are absent. He undergoes surgery, and a 27-cm segment of terminal ileum with a firm, erythematous serosal surface is removed. The microscopic appearance of a section through the excised ileum is shown. Which of the following additional complications is the patient most likely to develop as a result of this disease process?

Metastatic adenocarcinoma
Mesenteric artery thrombosis
Intussusception
Hepatic abscess
Enterocutaneous fistula
A

Enterocutaneous fistula

The ileum shows chronic inflammation with lymphoid aggregates. The inflammation is transmural, affecting the mucosa, submucosa, and muscularis. A deep fissure extending into the muscularis is also seen. These histologic features are highly suggestive of Crohn disease. Extension of fissures into the overlying skin can produce enterocutaneous fistulas, although enteroenteric fistulas between loops of bowel are more common.

Although the risk of adenocarcinoma is increased in Crohn disease, this complication is less common than sequelae of inflammation. Mesenteric artery thrombosis, typically a complication of atherosclerosis, is unlikely in a 27-year-old man. Intussusception may occur when there is a congenital or acquired obstruction in the bowel. Hepatic abscess can follow amebic colitis.

66
Q

A 59-year-old man has had increasing difficulty swallowing during the past 6 months. There are no significant findings on physical examination. Upper gastrointestinal endoscopy shows areas of erythematous mucosa above the Z-line. A biopsy specimen from the lower esophagus has the microscopic appearance shown. As a consequence of this patient’s condition, which of the following complications is most likely to occur?

Hematemesis
Squamous cell carcinoma
Adenocarcinoma
Achalasia
Lacerations (Mallory-Weiss syndrome)
A

Adenocarcinoma

The biopsy specimen shows columnar metaplasia, typical of Barrett esophagus. Patients with a focus of Barrett esophagus larger than 2 cm are at 30- to 40-fold higher risk of developing adenocarcinoma than is the general population.

Squamous cell carcinomas occur in the esophagus, but they do not arise in association with Barrett esophagus. Their occurrence is related to smoking and alcohol consumption. Hematemesis is a complication of esophageal varices and other conditions such as peptic ulcers. Achalasia refers to failure of relaxation of the lower esophageal sphincter that gives rise to dilation of the proximal portion of esophagus. Mallory-Weiss syndrome is associated with vertical lacerations in the esophagus that may occur with severe vomiting and retching.

67
Q

A 49-year-old man has complained of “heartburn” after meals for the past decade. There are no remarkable findings on physical examination. Upper gastrointestinal endoscopy is performed, and an esophageal biopsy specimen is taken from an erythematous area of velvety mucosa just above the gastroesophageal junction. Microscopically, the mucosa shows columnar metaplasia with goblet cells. Which of the following most likely produced these findings?

Oesophageal varices
Radiation therapy
Achalasia
Gastroesophageal reflux disease
Iron deficiency anemia
A

GORD

Columnar metaplasia of the lower esophageal mucosa, also called Barrett esophagus, is a consequence of chronic gastroesophageal reflux disease. The metaplasia may be accompanied by inflammation. Varices result from long-standing portal hypertension; although there may be associated inflammation, columnar metaplasia is not present. Irradiation may produce inflammation and eventual fibrosis, but there is no columnar metaplasia. Achalasia refers to failure of relaxation of the lower esophageal sphincter (LES). This gives rise to progressive dilation of the esophagus above the level of LES. Upper esophageal webbing may rarely accompany iron deficiency.

68
Q

A 44-year-old man has developed increasing arthritis pain, swelling of the feet, and reduced exercise tolerance over the past 3 years. He has smoked 1 pack of cigarettes per day for 20 years. Laboratory studies include serum glucose of 201 mg/dL, creatinine 1.1 mg/dL, and ferritin 893 ng/mL. A chest radiograph shows bilateral pleural effusions, pulmonary edema, and cardiomegaly. He undergoes a liver biopsy, and the microscopic appearance of a biopsy specimen stained with Prussian blue is shown. Based on these findings, which of the following is the most appropriate advice to give this patient?

A) You need to markedly reduce your alcohol consumption
B) A cholecystectomy should be performed
C) Your siblings may be at risk of developing the same condition
D) You will most likely develop acute fulminant hepatitis
E) Smoking for many years has led to this condition

A

Your siblings may be at risk of developing the same condition.

This patient has clinical, histologic, and laboratory features of genetic hemochromatosis. In this condition, iron overload occurs because of excessive absorption of dietary iron. The absorbed iron is deposited in many tissues, including the heart, pancreas, and liver, giving rise to heart failure, diabetes, and cirrhosis. It appears blue with Prussian blue stain, as seen in this figure. High serum ferritin concentration is an indicator of a vast increase in body iron. Genetic hemochromatosis is an autosomal recessive condition; hence, siblings are at risk of developing the same disease.

69
Q

A 53-year-old man comes to the emergency department because of marked hematemesis that has continued for the past 3 hours. On physical examination, he has a temperature of 35.9°C, pulse 112/min, respirations 26/min, and blood pressure 90/45 mm Hg. He has a distended abdomen with a fluid wave, and the spleen tip is palpable. Which of the following liver diseases is most likely to be present in this patient?

Cirrhosis
Cholangiocarcinoma
Massive hepatic necrosis
Fatty change

A

Cirrhosis

The findings point to portal hypertension with bleeding esophageal varices. Cirrhosis alters hepatic blood flow to produce portal hypertension.

A mass lesion, such as a cholangiocarcinoma, is unlikely to obstruct blood flow in this manner, nor does massive necrosis. Fatty change can increase liver size and can be associated with alcoholic cirrhosis, but steatosis alone does not elevate portal venous pressure. HAV infection rarely results in significant chronic liver disease.

70
Q

A 28-year-old man has had increasing shortness of breath for the past year. On physical examination, he is afebrile and normotensive. Breath sounds are decreased in all lung fields. His medical history indicates that he developed marked icterus as a neonate, but he been healthy since then. Because of a family history of liver disease, a liver biopsy is performed. The microscopic appearance of the liver biopsy specimen stained with PAS is shown. This patient is most likely at a very high risk for development of which of the following conditions?

Diabetes mellitus
Congestive heart failure
Pulmonary emphysema
Ulcerative colitis
Systemic lupus erythematosus
A

Pulmonary emphysema

The PAS-positive globules in the liver seen here are characteristic of α1-antitrypsin (AAT) deficiency. Approximately 10% of persons with the homozygous deficiency (PiZZ phenotype) of AAT deficiency will develop significant liver disease, including neonatal hepatitis and progressive cirrhosis. Deficiency of AAT also allows unchecked action of elastases in the lung, which destroys the elastic tissue and causes emphysema.

Diabetes mellitus and heart failure are features of hemochromatosis, a condition of iron overload. Iron deposition in liver is detected by the Prussian blue stain. Ulcerative colitis is strongly associated with primary sclerosing cholangitis, a condition in which there is inflammation and obliterative fibrosis of bile ducts. Systemic lupus erythematosus is an immune complex disease that may affect many organs. Liver involvement, however, is uncommon.

71
Q

A 41-year-old woman experienced increasing malaise and a 10-kg weight loss in the last year of her life. She became increasingly obtunded, lapsed into a coma, and died. At autopsy, the liver has the gross appearance shown. Ingestion of which of the following substances is most likely to have played a role in the development of this condition?

Aflatoxins
Raw oysters
Aspirin
Ferrous sulfate
Nitrites
Acetaminophen
A

Aflatoxin

Aflatoxin is a hepatotoxin and is the product of the fungus Aspergillus flavus, which grows on moldy peanuts. Aflatoxin can be carcinogenic, leading to hepatocellular carcinoma, as shown in the figure. Oysters can concentrate HAV from seawater contaminated with sewage, and eating raw oysters can result in HAV infection. Aspirin has been implicated in causing Reye syndrome in children, which results in extensive microvesicular steatosis. Prolonged and excessive intake of oral iron, rarely, can cause secondary hemochromatosis. Nitrites have been causally linked with cancers in the upper gastrointestinal tract. Ingestion of large amounts of acetaminophen leads to hepatocellular necrosis.

72
Q

A 48-year-old man sees his physician because he has had nausea and colicky right upper quadrant pain for the past 2 days. On physical examination, his temperature is 38.8°C. Laboratory studies show a WBC count of 11,200/mm3 with 71% segmented neutrophils, 9% bands, 13% lymphocytes, and 7% monocytes. Which of the following is the most likely diagnosis?

Acute HAV infection
Extrahepatic biliary atresia
Acute cholecystitis
Primary sclerosing cholangitis
Adenocarcinoma of the gallbladder
A

Acute cholecystitis

The symptoms are typical of acute calculous cholecystitis. Hepatitis is unlikely to produce acute pain and leukocytosis. Extrahepatic biliary atresia occurs in neonates and is characterized by obstructive jaundice. Sclerosing cholangitis is typically a chronic process that presents with jaundice and pruritus. Carcinomas of the gallbladder are not common and typically have a more insidious onset.

73
Q

A 51-year-old man has a lengthy history of chronic alcoholism and comes to the physician because of increasing malaise for the past year. He was hospitalized 1 year ago because of upper gastrointestinal hemorrhage. Physical examination shows a firm nodular liver. Laboratory findings show a serum albumin level of 2.5 g/dL and prothrombin time of 28 sec. Which of the following additional physical examination findings is most likely to be present?

Splinter hemorrhages
Diminished deep tendon reflexes
Caput medusae
Papilledema
Distended jugular veins
A

Caput medusae

This patient has alcoholic cirrhosis with portal hypertension. Venous collateral flow can be increased in esophageal submucosal veins, producing varices, and in the abdominal wall, producing caput medusae.

The coagulopathy from decreased liver function may lead to purpuric hemorrhages, but splinter hemorrhages of the nails are most characteristic of embolization from infective endocarditis. Liver failure with cirrhosis may lead to hepatic coma, but brain swelling with papilledema is not a major feature. Hyperreflexia, but not diminution of deep tendon reflexes, can occur when hepatic encephalopathy develops. Right-sided heart failure, in which the liver may be enlarged because of passive congestion, is associated with distended jugular veins.

74
Q

A 30-year-old woman has noted a 5-kg weight gain over the past 3 months and has not had a menstrual period during that time. She has experienced upper abdominal pain for the past month. Physical examination shows abdominal enlargement with apparent ascites. There is no peripheral edema. She has a positive pregnancy test. Additional laboratory findings show hemoglobin of 13.2 g/dL, hematocrit 39.7%, WBC count 12,300/mm3, glucose 80 mg/dL, AST 581 U/L, ALT 611 U/L, total bilirubin 1.3 mg/dL, total protein 6.2 g/dL, and albumin 3.5 g/dL. An abdominal ultrasound scan shows hepatomegaly with heterogenous echogenicity, and there is an intrauterine gestation with a fetus estimated at 12 weeks’ size. Which of the following pathologic findings is most likely to be present?

Choledocholithiasis
Chronic passive congestion
Hepatic venous thrombosis
Hepatocellular adenoma
Metastatic choriocarcinoma
Microvesicular steatosis
A

Hepatic venous thrombosis

The patient has Budd-Chiari syndrome, a rare condition that can complicate pregnancy or the postpartum state. Hepatic venous occlusion leads to hepatomegaly with severe centrilobular congestion and necrosis, much more pronounced than the typical “nutmeg” liver of chronic passive congestion with right-sided heart failure. Biliary tract obstruction with choledocholithiasis would increase the serum bilirubin to a greater degree than in this patient, and hepatomegaly is not likely. Hepatic adenomas, which can be associated with use of oral contraceptives, are mass lesions, usually several centimeters in size. At 12 weeks’ gestation, with a fetus present, choriocarcinoma is very unlikely, and a marked increase in liver enzymes is not likely. Acute fatty liver of pregnancy with microvesicular steatosis produces a more uniform density with hepatomegaly, a condition that is seen in the third trimester of pregnancy.

75
Q

For the past 24 years, a 70-year-old man has had hemoglobin A1c values between 8% and 11%. On physical examination, he has hard exudates and cotton-wool spots, as well as foci of neovascularization, seen on funduscopy. He now has increasing serum urea nitrogen and creatinine levels as well as proteinuria. Which of the following pathologic findings on renal biopsy is most likely to be present?

Membranous glomerulonephritis
Amyloid deposition
Diffuse glomerulosclerosis
Necrotizing vasculitis
Polycystic change
A

Diffuse glomerulosclerosis

Diffuse glomerulosclerosis and nodular glomerulosclerosis are changes that are characteristic of diabetic nephropathy. The nephropathy takes years to develop, and renal function gradually diminishes. The other complication of diabetes mellitus in this man is retinopathy. The hard exudates and cotton-wool spots are part of a background retinopathy, but the neovascularization is part of the more ominous proliferative retinopathy.

Membranous glomerulonephritis is most often idiopathic. Amyloidosis is uncommonly associated with diabetes mellitus. Although there are progressive vascular changes in the kidney with diabetes mellitus—notably, large-artery atherosclerosis and hyaline arteriolosclerosis—vasculitis is not a feature of diabetes mellitus. Polycystic changes are most likely to appear in diabetic patients who have received long-term hemodialysis

76
Q

One week after a mild flulike illness, a 9-year-old boy has an episode of hematuria that subsides within 2 days. One month later, he tells his parents that his urine is red again. On physical examination, there are no significant findings. Urinalysis shows a pH of 7, specific gravity 1.015, 1+ proteinuria, 1+hematuria, and no ketones, glucose, or urobilinogen. The serum urea nitrogen level is 36 mg/dL, and the creatinine level is 3.2 mg/dL. A renal biopsy specimen shows diffuse mesangial proliferation and electron-dense deposits in the mesangium. Which of the following mechanisms is most likely to produce these findings?

A) Deposition of immune complexes containing IgA
B) Formation of antibodies against type IV collagen
C) Virus-mediated injury to the glomeruli
D) Cytokine-mediated injury to the glomerular capillaries
E) Congenital defects in the structure of glomerular basement membranes

A

A

Development of recurrent hematuria following a viral illness in a child or young adult is typically associated with IgA nephropathy. In these patients, some defect in immune regulation causes excessive mucosal IgA synthesis in response to viral or other environmental antigens. IgA complexes are deposited in the mesangium and initiate glomerular injury. Antibodies against type IV collagen are formed in Goodpasture syndrome. Although viruses induce IgA synthesis, they do not cause direct glomerular damage. Cytokine-mediated injury can occur in transplant rejection. Defects in the structure of glomerular basement membrane are a feature of hereditary nephritis.

77
Q

A 49-year-old man saw his physician because he had increased swelling in the extremities for 2 months. Physical examination showed generalized edema. A 24-hour urine collection yielded 4.1 g of protein as well as albumin and globulins. Extensive testing did not indicate the presence of a systemic disease, such as diabetes mellitus or systemic lupus erythematosus. He did not respond to a course of corticosteroid therapy. A renal biopsy was performed, and microscopic examination showed diffuse thickening of the basement membrane. Immunofluorescence staining with antibody to the C3 component of complement was positive in a granular pattern in the glomerular capillary loops. Two years later, he experiences increasing malaise. Laboratory studies now show serum creatinine level of 4.5 mg/dL and urea nitrogen level of 44 mg/dL. Which of the following immunologic mechanisms was most likely responsible for the glomerular changes observed in the biopsy specimen?

Antibodies that react with basement membrane collagen
Antibodies against streptococci that cross-react with the basement membrane
Release of cytokines by inflammatory cells
Cytotoxic T cells directed against renal antigens
Deposition of immune complexes on the basement membrane

A

Deposition of ICs on the BM

This patient has idiopathic membranous glomerulopathy, producing nephrotic syndrome. Diffuse basement membrane thickening, in the absence of proliferative changes, and granular deposits of IgG and C3 are typical of this condition. It is caused by the deposition of immune complexes on the basement membrane, which activates complement.

Antibodies that react with basement membrane give rise to a linear immunofluorescence pattern, as in Goodpasture syndrome. Membranous glomerulopathy has no association with streptococcal infections. There is also no evidence of cytokine- or T-cell–mediated damage in this disease. In 85% of patients with membranous glomerulopathy, the cause of immune complex deposition is unknown. In the remaining 15%, an associated systemic disease (e.g., systemic lupus erythematosus) or some known cause of immune complex formation (e.g., drug reaction) exists

78
Q

Several members of a family developed chronic renal failure by the age of 50 years. Most are males. The affected persons also developed visual problems. Some younger family members have proteinuria and hematuria on urinalysis. A renal biopsy specimen from a 20-year-old man shows prominent tubular foam cells and glomerular basement membrane thickening and thinning. Family members with this disease are most likely to have which of the following additional manifestations?

Watery diarrhea
Nerve deafness
Presenile dementia
Dilated cardiomyopathy
Infertility
A

SNHL

These findings are characteristic of Alport syndrome, a form of hereditary nephritis. Most cases are inherited in an X-linked dominant pattern, but autosomal dominant and autosomal recessive patterns of inheritance also are seen. Most commonly, males are severely affected. Vision, hearing, and renal function are affected, but other organ systems are not affected.

79
Q

A 61-year-old woman sees the physician because she has experienced increasing malaise for the past 5 years. On physical examination, there are no abnormalities other than a blood pressure of 150/95 mm Hg. One week later, she dies suddenly and unexpectedly. At autopsy, both kidneys have the external (left panel) and bisected (right panel) appearance, as shown = polycystic. Which of the following conditions was the most probable cause of death?

Metastatic Wilms tumor
Ruptured berry aneurysm
Acute tubular necrosis
Disseminated intravascular coagulation
Pneumothorax
A

Ruptured berry aneurysm

These findings are characteristic of autosomal dominant polycystic kidney disease (DPKD). As seen in the figure, several large cysts have completely replaced the kidney. In autosomal recessive polycystic kidney disease, which typically presents in fetal and neonatal life, the kidneys have a smooth external appearance. On cut section, many small cysts give the kidney a spongelike appearance. About 10% to 30% of affected patients with DPKD have an intracranial berry aneurysm, and some of these can rupture without warning. Wilms tumor does not arise in a polycystic kidney. Acute tubular necrosis is the result of ischemic or toxic renal injuries. Disseminated intravascular coagulation may complicate hemolytic-uremic syndrome. Pulmonary disease does not accompany adult polycystic kidney disease.

80
Q

A 30-year-old woman with a history of recurrent urinary tract infections has had a high fever for the past 3 days. On physical examination, her temperature is 38.4°C. There is marked abdominal tenderness on deep palpation. A renal sonogram shows an enlarged right kidney with pelvic and calyceal enlargement and cortical thinning; the left kidney appears normal. A right nephrectomy is performed, and microscopic examination shows inflammatory infiltrates extending from the medulla to the cortex, with tubular destruction, glomerulosclerosis, and extensive interstitial fibrosis. Lymphocytes, plasma cells, and neutrophils are abundant. Which of the following is most likely to produce these findings?

Benign nephrosclerosis
Vesicoureteral reflux
Lupus nephritis
Systemic amyloidosis
Congestive heart failure
Autosomal dominant polycystic kidney disease
A

Vesico-ureteric reflux

These changes are characteristic of chronic pyelonephritis. Urinary tract obstruction favors recurrent urinary tract infection. Vesicoureteral reflux propels infected urine from the urinary bladder to the ureters and renal pelvis and predisposes to infection.

Benign nephrosclerosis is a vascular disease that does not carry a risk of infection. Lupus nephritis is associated with extensive inflammatory changes of glomeruli that are noninfectious. Amyloidosis can lead to progressive renal failure as a result of amyloid deposition in the glomeruli; however, amyloid does not evoke an inflammatory response. Congestive heart failure may predispose to acute tubular necrosis. Autosomal dominant polycystic kidney disease is a bilateral process; patients usually are not symptomatic until middle age

81
Q

Over the past 72 hours, a 44-year-old man has experienced worsening headache, nausea, and vomiting. On physical examination, his blood pressure is 276/158 mm Hg, and there is bilateral papilledema. Urinalysis shows 2+ proteinuria, 1+ hematuria, and no glucose or ketones. Which of the following renal lesions is most likely to be present in this patient?

Papillary necrosis
Acute infarction
Necrotizing arteriolitis
Acute tubular necrosis
Acute pyelonephritis
A

Necrotizing arteriolitis

This patient has malignant hypertension. Necrotizing arteriolitis and hyperplastic arteriolosclerosis are the distinctive vascular lesions of malignant hypertension. Papillary necrosis is more likely to complicate diabetic nephropathy or analgesic nephropathy. Infarction of the kidney may result from emboli originating in the systemic circulation. However, malignant hypertension does not damage the large systemic vessels. Acute tubular necrosis is seen in hypoxic or toxic injury to the renal tubules. Acute pyelonephritis is a febrile illness, without severe blood pressure elevation.

82
Q

Several days after eating a hamburger, chili, and ice cream at a home barbecue, a 5-year-old girl develops cramping abdominal pain and diarrhea. The next day, she has decreased urine output. On physical examination, there are petechial hemorrhages on the skin. Her temperature is 37.0°C, pulse 90/min, respirations 18/min, and blood pressure 90/50 mm Hg. A stool sample is positive for occult blood. Laboratory findings show hemoglobin of 10.8 g/dL, hematocrit 32.4%, platelet count 64,300/mm3, and WBC count 6480/mm3. The peripheral blood smear shows schistocytes, and the serum D dimer level is elevated. Which of the following is the most likely causative organism?

Candida albicans
Proteus mirabilis
Clostridium difficile
Escherichia coli
Staphylococcus aureus
A

E.coli

This girl has hemolytic-uremic syndrome. Some strains of Escherichia coli, which can contaminate ground beef products, may elaborate a toxin that damages endothelium, causing this syndrome. Hemolytic-uremic syndrome most often occurs in children and is one of the most common causes of acute renal failure in children. Candidal urinary tract infections typically affect the urinary bladder. Proteus is a common cause of bacterial urinary tract infections. Clostridium difficile is best known for causing a pseudomembranous enterocolitis, not renal lesions. Staphylococcus aureus can cause urinary tract infections.

83
Q

A 62-year-old childless woman noticed a blood-tinged vaginal discharge twice during the past month. Her last menstrual period was 14 years ago. Bimanual pelvic examination shows that the uterus is normal in size, with no palpable adnexal masses. There are no cervical erosions or masses. Her medical history indicates that for the past 30 years she has had hypertension and type 1 diabetes mellitus, which is treated with insulin injections. An endometrial biopsy is most likely to show which of the following?

Adenomyosis
Leiomyosarcoma
Adenocarcinoma
Squamous cell carcinoma
Choriocarcinoma
Malignant mixed müllerian tumor
A

Adenocarcinoma

Postmenopausal vaginal bleeding is a “red flag” for endometrial carcinoma. Such carcinomas often arise in the setting of endometrial hyperplasia. Increased estrogenic stimulation is thought to drive this process, and risk factors include obesity, diabetes mellitus, hypertension, and infertility. Adenomyosis is an extension of endometrial glands and stroma into the myometrium, generally resulting in symmetric uterine enlargement. A submucosal leiomyosarcoma could produce vaginal bleeding, but the uterus would be enlarged, because leiomyosarcomas tend to be large masses. Squamous carcinomas of the endometrium are rare. Choriocarcinomas are gestational in origin. Malignant mixed müllerian tumors are much less common than endometrial carcinomas, but they could produce similar findings.

84
Q

A 42-year-old woman has a Pap smear as part of a routine health maintenance examination. Her previous Pap smear was obtained 10 years ago. There are no remarkable findings on physical examination. The current Pap smear shows findings consistent with cervical intraepithelial neoplasia (CIN) III. A cervical biopsy is performed, and microscopic examination shows dysplastic cells that occupy the full thickness of the epithelium above the basement membrane. Which of the following is the most likely explanation for proceeding with cervical conization for this patient?

She has a high risk of invasive carcinoma
Human papillomavirus infection cannot be treated
She is perimenopausal
CIN I is present
She has invasive cancer

A

She is a high risk of invasive carcinoma

The patient has cervical intraepithelial neoplasia (CIN) III, or carcinoma in situ, which may progress to invasive carcinoma in several years if not treated. Infection with human papillomavirus (HPV) often drives this process, but the presence of HPV alone does not determine therapy. HPV infection cannot be eradicated, but the goal of eradication is not the reason to treat dysplasias. A dysplasia requires treatment regardless of the patient’s age. A CIN I lesion involves only part of the thickness of the cervical epithelium. Invasive cancers penetrate the basement membrane; hence, the cancer cells are seen below the basement membrane

85
Q

A 40-year-old woman has noticed progressive enlargement of the abdomen over the past 5 months, although her diet has not changed and she has been exercising more. Physical examination shows no palpable masses, but a fluid wave is present. Paracentesis yields 500 mL of slightly cloudy fluid. Cytologic examination of the fluid shows malignant cells. An abdominal ultrasound scan shows a 15-cm multilobular mass that involves the right adnexal region. The uterus is normal in size. The mass is surgically removed, and the gross features of a section of the excised mass are shown: polycystic mass. Which of the following is the most likely diagnosis?

Immature teratoma
Mucinous cystadenocarcinoma
Granulosa cell tumor
Choriocarcinoma
Dysgerminoma
A

Mucinous cystadenocarcinoma

Mucinous tumors of the ovary are of epithelial origin, are less common than serous tumors, and tend to be multiloculated. The appearance of ascites suggests metastases, which is most common with surface epithelial neoplasms of the ovary. Immature teratomas tend to be solid masses, as do granulosa cell tumors and dysgerminomas. Choriocarcinomas rarely reach this size, because they metastasize early; they are typically hemorrhagic.

86
Q

A 47-year-old woman has had increasing abdominal enlargement, with no significant pain, and diarrhea for the past 3 months. She goes to the physician, who performs a cursory physical examination and obtains a stool culture that is negative. The diagnosis is irritable bowel syndrome. She continues to have increasing abdominal enlargement over the next month, reaching the size of a 5-month pregnancy. She sees another physician, who finds a fluid wave on examination of the abdomen. An abdominal CT scan shows massive ascites and scattered 0.5- to 1.5-cm cystic to solid nodules on the surfaces of the bowel and abdominal wall. Paracentesis yields a yellow, slightly cloudy fluid with a high protein content. Cytologic examination of the fluid shows clusters of malignant cells. Laboratory studies show a positive CA-125 and a negative carcinoembryonic antigen test result. The patient’s medical history indicates that she underwent a total abdominal hysterectomy with salpingo-oophorectomy 10 years ago. Which of the following is the most likely diagnosis?

Adenocarcinoma of the ileum
Carcinoid tumor
Endometrioid carcinoma
Malignant mesothelioma
Metastatic ovarian serous cystadenocarcinoma
Mucinous cystadenoma
Serous peritoneal carcinoma
A

Serous peritoneal carcinoma

A serous peritoneal carcinoma has gross and microscopic characteristics, as well as immunohistochemical features and serum tumor markers, that are identical to an ovarian serous cystadenocarcinoma, but it is thought to arise from the mesothelium, probably in a multifocal fashion. It is unlikely, even if the patient had a prior ovarian neoplasm, that it lay dormant for 10 years. The risk of serous peritoneal carcinoma is increased in women who have had an ovarian serous cystadenocarcinoma. Ileal adenocarcinomas are rare and would probably lead to bowel obstruction. Carcinoid tumors are unlikely to be widely metastatic. Endometrioid carcinomas arise in the ovary. A malignant mesothelioma is a rare complication of asbestosis. It is unlikely that a cystadenocarcinoma would recur 10 years later. Cystadenomas are benign.

87
Q

A 28-year-old woman in the third trimester of her third pregnancy discovered a lump in her right breast. The physician palpated a 2-cm discrete, freely movable mass beneath the nipple. After the birth of a term infant, the mass appears to decrease slightly. The infant breast-feeds without difficulty. Which of the following is the most likely diagnosis?

Intraductal papilloma
Phyllodes tumor
Lobular carcinoma in situ
Fibroadenoma
Medullary carcinoma
A

Fibroadenoma

Fibroadenomas are common and may enlarge during pregnancy or late in each menstrual cycle. Most intraductal papillomas are smaller than 1 cm and are not influenced by hormonal changes.

Phyllodes tumors are uncommon and tend to be larger than 4 cm. Lobular carcinoma in situ is typically an ill-defined lesion without a mass effect. Medullary carcinomas tend to be large; they account for only about 1% of all breast carcinomas.

88
Q

A 35-year-old woman feels a poorly defined lump in her right breast. The physician palpates an irregular 1-cm mass in the upper inner quadrant. The mass is not painful and does not feel firm. There are no lesions of the overlying skin and no axillary lymphadenopathy. The patient has normal menstrual cycles. She is G 3, P 3, and her last child was born 5 years ago. Which of the following is the most likely diagnosis?

Fibrocystic changes
Lobular carcinoma
Acute mastitis
Fibroadenoma
No pathologic diagnosis
A

Fibrocystic changes

Statistically, the largest category of breast “lumps” (40% of all masses) comprises fibrocystic changes. These lesions are probably related to cyclic breast changes that occur during the menstrual cycle. In about 30% of cases, no specific pathologic diagnosis can be made. Carcinomas often produce palpable masses and represent about 10% of lumps. Fibroadenomas constitute 7% of lumps, but they are usually discrete, firm masses. Acute mastitis does not usually manifest as a lump and is most often a complication of lactation.

89
Q

A 30-year-old man comes to his physician because he has noticed increasing enlargement and a feeling of heaviness in his scrotum for the past year. On physical examination, the right testis is twice its normal size, and it is firm and slightly tender. An ultrasound examination shows a 3.5-cm solid mass in the right testis. An abdominal CT scan shows enlargement of the para-aortic lymph nodes. Multiple lung nodules are seen on a chest radiograph. Laboratory findings include markedly increased serum levels of chorionic gonadotropin and α-fetoprotein. Which of the following testicular neoplasms is the most likely diagnosis?

Leydig cell tumor
Mixed germ cell tumor
Pure spermatocytic seminoma
Choriocarcinoma
Metastatic adenocarcinoma of the prostate gland
Large diffuse B-cell lymphoma
A

Mixed germ cell tumour

Although a modest elevation of the human chorionic gonadotropin (hCG) concentration can occur when a seminoma contains some syncytial giant cells, significant elevation of the α-fetoprotein (AFP) level never occurs with pure seminomas. Elevated levels of AFP and hCG effectively exclude the diagnosis of a pure seminoma and indicate the presence of a nonseminomatous tumor of the mixed type. The most common form of testicular neoplasm combines multiple elements; thus the term “teratocarcinoma” is sometimes used to describe tumors with elements of teratoma, embryonal carcinoma, and yolk sac tumor. The yolk sac element explains the high AFP level. Mixed tumors may include seminoma. Leydig cell tumors are non–germ cell tumors derived from the interstitial (Leydig) cells; they may elaborate androgens.

Choriocarcinomas secrete high levels of hCG but no AFP. It is unusual for a tumor to metastasize to the testis; this patient is of an age at which a primary cancer of the testis should be considered when a testicular mass is present. Lymphomas may involve the testis, usually when there is systemic involvement by a high-grade lesion. Lymphomas do not elaborate hormones.

90
Q

An infant is born at term to a 41-year-old woman after an uncomplicated pregnancy. Soon after birth, the neonate develops hypotension. Physical examination shows no anomalies. Laboratory studies show Na+ of 131 mmol/L, K+ 5.1 mmol/L, Cl− 93 mmol/L, CO2 18 mmol/L, glucose 65 mg/dL, creatinine 0.4 mg/dL, testosterone 50 ng/dL (normal

A

21-hydroxylase

A deficiency of 21-hydroxylase leads to a salt-wasting form of adrenogenital syndrome, because the enzyme deficiency blocks formation of aldosterone and cortisol. A deficiency of 11-hydroxylase blocks cortisol and aldosterone production as well, although intermediate metabolites with some glucocorticoid activity also are synthesized.

Aromatase is involved with conversion of androstenedione to estrone, a pathway of steroid synthesis that does not affect cortisol production. A deficiency of 17α-hydroxylase would lead to reduction of cortisol and sex steroid synthesis. Oxidase is the final enzyme in the pathway to aldosterone production

91
Q

A 20-year-old woman and her twin sister both experience increasing diplopia. Their conditions develop within 3 years of each other. On physical examination, they have exophthalmos and weak extraocular muscle movement. The thyroid gland is diffusely enlarged but painless in each sister, and there is no lymphadenopathy in either woman. Which of the following laboratory findings is most likely to be reported in these sisters?

Decreased serum free T4 level
Increased urinary free catecholamine level
Decreased serum thyroid-stimulating hormone level
Increased serum parathormone level
Increased serum thyrotropin-releasing hormone level
High titer serum antimicrosomal antibody

A

Decressed serum TSH level

Exophthalmos is a feature seen in about 40% of persons with Graves disease. The hyperfunctioning thyroid gland leads to an increased T4 level, with positive feedback from the pituitary to decrease TSH secretion. There is about 50% concordance of Graves disease among identical twins. The autoimmune character of this disorder is evidenced by an association with HLA-DR3. A decreased T4 level is indicative of hypothyroidism. Increased urinary free catecholamines are seen in pheochromocytomas. An increased parathormone level is seen in parathyroid adenomas and carcinomas. An increased thyrotropin-releasing hormone (TRH) level would increase the TSH level and increase the T4, but feedback typically occurs at the level of both the pituitary and the hypothalamus, and abnormal increases in TRH are uncommon. Antithyroid peroxidase antibodies can be seen in Hashimoto thyroiditis and in Graves disease, but the highest titers occur in Hashimoto thyroiditis.

92
Q

A 50-year-old woman has been bothered by a discolored area of skin on her forehead that has not faded during the past 3 years. On physical examination, there is a 0.8-cm red, rough-surfaced lesion on the right forehead above the eyebrow. A biopsy specimen examined microscopically shows hyperkeratosis and thinning of the epidermis with atypical basal cells. The upper dermal collagen and elastic fibers show homogenization with elastosis. Which of the following is the most appropriate advice to give this patient?

Reduce intake of dietary fat
Wear a hat outdoors
Stop taking aspirin for headaches
Apply hydrocortisone cream to your face
This condition is related to aging
A

Wear a hat outdoors

Actinic keratoses are premalignant lesions associated with sun exposure. Decreasing dietary fat is always a good idea, but it does not have much effect on the skin of the face. Many drugs can cause acute eczematous dermatitis and erythema multiforme. Hydrocortisone can alleviate the symptoms of many dermatologic conditions, but it cannot reverse actinic damage. Older persons are more likely to have actinic keratoses because of greater cumulative sun exposure, not because of aging alone.

93
Q

A 49-year-old woman has been bothered for at least 20 years by recurring skin lesions that are most prominent over the elbows and knees and also sometimes on the scalp and lumbosacral area. These skin lesions are silvery to salmon-colored 1- to 4-cm plaques with scaling. The lesions seem to form more readily at sites of minor trauma, such as a superficial abrasion. She has had increasing pain in her left hand and in her hips, more prominent on the left, over the past 2 years. On physical examination, she has yellow-brown discoloration with pitting of the fingernails. The distal interphalangeal joints of digits two and three of the left hand are slightly swollen and tender. There is minimal reduction in left hip mobility and no swelling or warmth to the touch. A radiograph of the left hip shows minimal joint space narrowing and surface erosion. Bone density is not markedly reduced. During the next 10 years, the joint pain persists, but there is no joint destruction or deformity. She continues to have the same skin lesions. Which of the following is most likely to be seen on biopsy of these skin lesions?

Bandlike upper dermal infiltrate of lymphocytes
Epidermal spongiosis with dermal edema and eosinophils
Focal keratinocyte apoptosis
Hyperkeratosis with parakeratosis and elongated rete ridges
IgG deposited at the dermal-epidermal junction

A

Hyperkeratosis with parakeratosis and elongated rete ridges

The patient has psoriasis with psoriatic arthritis, which has features of rheumatoid arthritis but without significant joint destruction. Psoriasis is common, affecting 1% to 2% of persons, and about 5% of these have psoriatic arthritis. For the remaining choices, there is no significant association with arthritis. A bandlike dermal infiltrate is typical of lichen planus, which produces pruritic violaceous plaques or papules but tends to abate in 1 to 2 years. Epidermal spongiosis with eosinophilic infiltrates can be seen in acute eczematous dermatitis as part of a drug reaction. Focal keratinocyte apoptosis is seen in graft-versus-host disease. IgG deposition can be seen in systemic lupus erythematosus and in bullous pemphigoid.

94
Q

A 55-year-old previously healthy man has had episodes of pain and swelling of the right first metatarsophalangeal (MP) joint for the past year. These flare-ups usually occur after consumption of alcohol, typically port wine (six grapes). On physical examination, there is exquisite tenderness with swelling and erythema of the right first MP joint. A joint aspiration is performed, and laboratory studies of the fluid obtained show needle-shaped crystals and many neutrophils in a small amount of fluid. Which of the following laboratory findings is most likely to be reported in this man?

Increased serum parathyroid hormone level
Elevated serum urea nitrogen level
Hyperuricemia
Markedly elevated levels of serum transaminases
Elevated rheumatoid factor titer

A

Hyperuricaemia

Acute inflammation of the first metatarsophalangeal joint, caused by precipitation of uric acid crystals in the joint space, is typical of gout. Hyperuricemia is a sine qua non for the development of gout. However, all patients with hyperuricemia do not develop gout. Other, ill-defined factors play a role in pathogenesis. Involvement of the big toe is classic, but other joints may be involved. An increased parathyroid hormone level indicates primary hyperparathyroidism, which is unlikely to produce joint disease. Elevated serum urea nitrogen is present in renal failure, which may produce secondary hyperparathyroidism. Although attacks of gout are often precipitated by a heavy bout of alcohol consumption, liver damage (marked by elevated transaminases) is not a feature of gouty arthritis. Gout is not associated with rheumatoid arthritis.

95
Q

A 35-year-old man has experienced increasing weakness of the pelvic and shoulder girdle muscles for several years. On physical examination, he has 4/5 motor strength in all extremities. There is no pain or deformity, and his range of motion is normal. He has no gait abnormality and no tremors. There are no focal neurologic deficits. He is afebrile. A deltoid biopsy is performed, and Western blot analysis shows reduced amounts of dystrophin with an abnormal molecular weight. Which of the following is the most likely diagnosis?

Amyotrophic lateral sclerosis
Becker muscular dystrophy
Duchenne muscular dystrophy
Lambert-Eaton syndrome
McArdle disease
Myasthenia gravis
Myotonic dystrophy
Polymyositis
Werdnig-Hoffmann disease
A

Becker’s MD

This patient has Becker muscular dystrophy. This congenital condition is similar to Duchenne muscular dystrophy in that it has an X-linked pattern of inheritance and there is a mutation in the dystrophin gene. However, in Becker muscular dystrophy, dystrophin is abnormal, not absent, resulting in less severe muscle disease than that of Duchenne muscular dystrophy. The typical patient is a middle-aged man.

Amyotrophic lateral sclerosis is a progressive disease with a neurogenic form of muscle atrophy resulting from loss of motor neurons. Lambert-Eaton syndrome is a myasthenia-like paraneoplastic condition. McArdle disease is caused by a deficiency in muscle phosphorylase and does not produce progressive weakness. Myasthenia gravis results from acetylcholine receptor antibody and leads to progressive weakness. Myotonic dystrophy is an X-linked disorder characterized by facial and upper body weakness, cataracts, gonadal atrophy, cardiomyopathy, and dementia. Polymyositis is an autoimmune disorder with myalgia. Werdnig-Hoffman disease is a spinal muscular atrophy resulting from loss of motor neurons in infancy.

96
Q

A 45-year-old previously healthy man has developed headaches over the past month. On physical examination, there are no remarkable findings. A cerebral angiogram shows a 7-mm saccular aneurysm at the trifurcation of the right middle cerebral artery. Which of the following is most likely to result from this lesion?

Epidural hematoma
Subarachnoid hemorrhage
Subdural hematoma
Cerebellar tonsillar herniation
Hydrocephalus
A

SAH

Intracranial aneurysms are typically saccular and slowly enlarge over time. Those that reach 4 to 7 mm are at greatest risk of rupture. Rupture occurs into the subarachnoid space at the base of the brain, where the cerebral arterial distribution originates around the circle of Willis and where saccular aneurysms are most likely to arise. Epidural hematomas arise from a tear of the middle meningeal artery, typically as a result of head trauma. Trauma can also cause a tear of bridging veins that produces a subdural hematoma. Neither a berry aneurysm nor the bleeding that results is likely to cause a mass effect and herniation. In some cases of survival after rupture of a berry aneurysm, a noncommunicating hydrocephalus results from organization of the subarachnoid hemorrhage.

97
Q

A 55-year-old man dies of aspiration pneumonia after a 6-year illness, characterized by progressive, symmetric muscular weakness. At autopsy, the brain and spinal cord appear normal on gross examination. Microscopic sections show gliosis in the motor cortex, pallor of the lateral corticospinal tracts, and neuronal loss in the anterior horns of the spinal cord. Which of the following is the most likely underlying cause of death?

Becker muscular dystrophy
Neurofibromatosis type 2
Guillain-Barré syndrome
Creutzfeldt-Jakob disease
Amyotrophic lateral sclerosis
A

Amyotrophic lateral sclerosis causes loss of upper motor neurons, leading to progressive muscular weakness from grouped atrophy of skeletal muscle fibers (denervation atrophy). Becker muscular dystrophy can produce progressive weakness in middle-aged men as a result of the decreased dystrophin level in muscle, but the CNS is not affected. In neurofibromatosis, mass lesions may be seen, including neurofibromas, schwannomas, meningiomas, and gliomas. Guillain-Barré syndrome is a form of rapidly ascending paralysis that occurs over weeks. Creutzfeldt-Jakob disease is a rapidly progressive form of dementia that results in death in less than 1 year in most cases

98
Q

A 79-year-old woman was driving her automobile when she had a sudden, severe headache. She drove to a service station, stopped the vehicle, and slumped over the wheel. She was taken to the emergency department, where she remained comatose and died 6 hours later. The gross appearance of the brain at autopsy is showing an abnormality of the Circle of Willis. Which of the following is the most likely diagnosis?

Hemorrhage in a glioblastoma multiforme
Thromboembolization with cerebral infarction
Rupture of the bridging veins
Rupture of a berry aneurysm
Hyaline arteriolosclerosis with hemorrhage

A

Rupture of a berry aneurysm

The figure shows an aneurysm in the circle of Willis and extensive subarachnoid hemorrhage. The hemorrhage occurs at the base of the brain, because berry aneurysms involve the circle of Willis and its branches. The vasospasm caused by the leakage of blood can lead to ischemia that further complicates the course.

A glioblastoma often has extensive necrosis and some hemorrhage, but the blood remains intraparenchymal. Thromboembolic infarctions can be hemorrhagic, because the embolus does not completely occlude the vessel, but the hemorrhage is localized. Rupture of bridging veins occurs at the vertex beneath the dura, producing a subdural hematoma. Hypertensive hemorrhages most often occur in the basal ganglia, the pons, or the cerebellum from rupture of small arteries in hyaline arteriolosclerosis, and the blood typically remains intraparenchymal.

99
Q

A 39-year-old man who has been infected with HIV for at least 8 years has received no antiretroviral therapy. He has had left-sided weakness for the past month. He sees the physician 1 day after experiencing a generalized seizure. On physical examination, he is afebrile. There is 4/5 motor strength in the left upper extremity. CT scan of the head shows no intracranial hemorrhage, but there is a midline shift. MR imaging of the brain shows a 4-cm mass in the region of the putamen near the right internal capsule, a 3-cm mass in the right centrum semiovale, and a 1-cm mass near the splenium of the corpus callosum. These masses are circumscribed and solid. A lumbar puncture is performed, and analysis of the CSF shows an elevated protein concentration and a normal glucose level. Cytologic examination shows large cells with large nuclei and scant cytoplasm that mark with CD19 but not with GFAP or cytokeratin. Which of the following is the most likely diagnosis

Cytomegalovirus encephalitis
Glioblastoma multiforme
Kaposi sarcoma
Large B-cell lymphoma
Progressive multifocal leukoencephalopathy
Toxoplasmosis
A

Large B cell lymphoma

Non-Hodgkin lymphomas, including large B-cell lymphomas, are not common in the brain, but they may be seen in immunocompromised persons, particularly those with AIDS. They tend to be multifocal.

Cytomegalovirus infection is common in AIDS, but it is unlikely to produce mass lesions. A glioblastoma multiforme can be a large infiltrative and destructive mass, but the cells are typically GFAP positive and CD19 negative. Kaposi sarcoma can occur in association with HIV infection, but CNS involvement is rare, and the cells are CD34 positive. Progressive multifocal leukoencephalopathy produces granular white matter lesions with large, bizarre oligodendrocytes infected with JC papovavirus. Toxoplasmosis can produce multiple mass lesions, but they are chronic abscesses that are filled with necrotic material and surrounded by gliosis; toxoplasma pseudocysts may be found in the abscesses.

100
Q

A 68-year-old woman has had decreased visual acuity for the past 5 years. She has no ocular pain. Her intraocular pressure is normal. Findings on funduscopic examination include arteriolar narrowing, flame-shaped hemorrhages, cotton-wool spots, and hard, waxy exudates. Which of the following is the most likely diagnosis?

Chronic hypertension
Retinitis pigmentosa
Advanced atherosclerosis
Diabetes mellitus
Cerebral edema
A

Chronic hypertension

Hypertensive retinopathy results from long-standing hypertension, with progressive changes that begin with generalized narrowing of the arterioles and proceed to the changes seen in this case. Retinitis pigmentosa is an inherited condition that may begin later in life (but usually earlier) and produces a waxy pallor of the optic disc. Arteriosclerotic retinopathy causes vascular changes, including arteriovenous nicking and hyaline arteriolosclerosis with “copper-wire” and “silver-wire” arterioles. A variety of findings are associated with diabetic retinopathy, including capillary microaneurysms, cotton-wool spots, arteriolar hyalinization, and more severe changes of proliferative retinopathy with neovascularization. Cerebral edema may result in papilledema.